Download as pdf or txt
Download as pdf or txt
You are on page 1of 81

(Physics) ELECTROSTATICS

DPP - 1
PROPERTIES OF CHARGE AND COULOMB'S LAW
Q.1 Two point charges q1 = 2 × 10−3 C and q 2 = −3 × 10−6 C are separated by a
distance x = 10 cm. Find the magnitude and nature of the force between the two
charges.
Q.2 Two point charges q1 = 20μC and
q 2 = 25μC are placed at (−1,1,1)m and
(3,1, −2)m, with respect to a coordinate system. Find the magnitude and unit vector along
electrostatic force on q 2 ?
Q.3 20 positively charged particles are kept fixed on the X-axis at points
x = 1 m, 2 m, 3 m, … . ,20 m. The first
particle has a charge 1.0 × 10−6 C, the second 8 × 10−6 C, the third 27 × 10−6 C and so on. Find
the magnitude of the electric force acting on a 1C charge placed at the origin.
Q.4 (i) Two charged particles having charge 4.0 × 10−6 C and mass 24 × 10−3 Kg each
are joined by an insulating string of length 1 m and the system is kept on a smooth horizontal
table. Find the tension in the string.
(ii) If suddenly string is cut then what is the acceleration of each particle?
(iii) Are they having equal acceleration?
Q.5 Two identical conducting spheres (of negligible radius), having charges of
opposite sign, attract each other with a force of 0.108 N when separated by 0.5 meter. The
spheres are connected by a conducting wire, which is then removed (when charge stops flowing),
and thereafter repel each other with a force of 0.036 N keeping the distance same. What were
the initial charges on the spheres?
Q.6 Two small spheres, each of mass 0.1gm and carrying same charge 10−9 C are
suspended by threads of equal length from the same point. If the distance between the centres
of the sphere is 3 cm, then find out the angle made by the thread with the vertical.
1
(g = 10 m/s 2 ) & tan−1 ( ) = 0.6∘
100

Q.7 The distance between two fixed positive charges 4e and e is ℓ. How should a third charge ' q ' be
arranged for it to be in equilibrium? Under what condition will equilibrium of the charge ' q ' be
stable (for displacement on the line joining 4e and e) or will it be unstable?
Q.8 Three charges, each of value q, are placed at the comers of an equilateral triangle.
A fourth charge Q is placed at the centre O of the triangle.
(a) If Q = −q, will the charges at corners start to move towards centre or away from it.
(b) For what value of Q at O will the charges remain stationary?
1
(Physics)

Q.9 Two charged particles A and B, each having a charge Q are placed a distance d
apart. Where should a third particle of charge q be placed on the perpendicular bisector of AB so
that it experiences maximum force? Also find the magnitude of the maximum force.
Q.10 A charged particle q1 is at position (2, −1,3). The electrostatic force on another charged particle
q 2 at (0,0,0) is:
q q
(A) 56πϵ
1 2
(2î − ĵ + 3k̂)
0
q1 q2
(B) 56√14πϵ (2î − ĵ + 3k̂)
0
q1 q2
(C) (ĵ − 2î − 3k̂)
56πϵ0
q q
(D) 56√14πϵ
1 2
(ĵ − 2î − 3k̂)
0

Q.11 Three charges +4q, Q and q are placed in a straight line of length ℓ at points at distance 0, ℓ/2
and ℓ respectively from one end of line. What should be the value of Q in order to make the net
force on q to be zero?
(A) −q (B) −2q
(C) −q/2 (D) 4q
Q.12 Two similar very small conducting spheres having charges 40μC and −20μC are some distance
apart. Now they are touched and kept at the same distance. The ratio of the initial to the final
force between them is:
(A) 8: 1 (B) 4: 1
(C) 1: 8 (D) 1: 1
Q.13 Two point charges placed at a distance r in air exert a force F on each other. The value of distance
R at which they experience force 4 F when placed in a medium of dielectric constant K = 16 is :
(A) r (B) r/4
(C) r/8 (D) 2r
Q.14 Three charges +Q, q, +Q are placed respectively, at distance, 0 d/2 and d from the origin, on the
x-axis. If the net force experienced by +Q, Placed at x = 0, is zero, then value of q is

(9th jan 1st shift 2019)

(A) +Q/4 (B) –Q/2 (C) +Q/2 (D) –Q/4


Q.15 Two charges, each equal to q, are kept at x = – a and x = a on the x–axis. A particle of mass m and
q
charge q0 = is placed at the origin. If charge q0 is given a small displacement (y < < a) along
2
the y–axis, the net force acting on the particle is proportional to (JEE MAINS - 2013)
1 1
(A) − (B) y (C) –y (D)
y y
2
(Physics)

Q.16 Two identical charged spheres suspended from a common point by two massless strings of
length l are initially a distance d(d < < l) apart because of their mutual repulsion. The charge
begins to leak from both the spheres at a constant rate. As a result the charges approach each
other with a velocity v. Then as a function of distance x between them (JEE MAINS - 2011)

(A) v  x –1/2 (B) v  x –1 (C) v  x 1/2 (D) v  x

Q.17 Two identical charged spheres are suspended by strings of equal lengths. The strings make an
angle of 30° with each other. When suspended in a liquid of density 0.8 g cm–3 , the angle
remains the same. If density of the material of the sphere is 1.6 g c–3 , the dielectric constant of
the liquid is (JEE MAINS - 2010)

(A) 1 (B) 4 (C) 3 (D) 2

Q.18 If gE an gM are are the accelerations due to gravity on the surfaces of the earth and the moon
respectively and if Millikan’s oil Drop experiment could be performed on the two surfaces, one
electronic charge on the moon
will find the ratio to be (JEE MAINS - 2007)
electronic chargeon themoon

(A) gM/gE (B) 1 (C) 0 (D) gE/gM

Q.19. Two spherical conductors A and B and having equal radii and carrying equal charges in them
repel each other with a force F when kept apart at some distance. A third spherical conductor
having same radius as that of B but uncharged is brought in contact with B, then brought in
contact with A and finally removed away from both. Then new force of repulsion between B and
A is (JEE MAINS - 2004)

(A) F/4 (B) 3F/4 (C) F/8 (D) 3F/8

Q.20 Two equal negative charge −q are fixed at points (0, −a) and (0, a) on y-axis. A positive charge
Q is released from rest at the point (2a, 0) on the x-axis. The charge Q will (IIT 1984)
(A) execute simple harmonic motion about the origin
(B) move to the origin remain at rest
(C) move to infinity
(D) execute oscillatory but not simple harmonic motion.
Q.21 A charge q is placed at the centre of the line joining two equal charges Q. The system of the
three charges will be in equilibrium if q is equal to (IIT 1987)
Q Q Q Q
(a) − 2 (b) − 4 (c) + 4 (d) + 2 .
3
(Physics)

Q.22 Four charges Q1 , Q2 , Q3 and Q4 of same magnitude are fixed along the x-axis at x = −2a, −a, +a
and +2a respectively. A positive charge q is placed on the positive y-axis at a distance b > 0.
Four options of the signs of these charges are given in List I. The direction of the forces on the
charge q is given in List II. Match List I with List II and select the correct answer using the code
given below the lists. (JEE ADV. 2012)

List I List II
P. Q1 , Q2 , Q3 , Q4 all positive 1. +x
Q. Q1 , Q2 positive; Q3 , Q4 negative 2. −x
R. Q1 , Q4 positive; Q2 , Q3 negative 3. +y
S. Q1 , Q3 positive; Q2 , Q4 negative 4. −y
Code :
(A) P − 3, Q − 1, R − 4, S − 2
(B) P − 4, Q − 2, R − 3, S − 1
(C) P − 3, Q − 1, R − 2, S − 4
(D) P − 4, Q − 2, R − 1, S − 3
23. Two small equally charged spheres, each of mass 𝑚, are suspended from the same point by silk
threads of length 1 . The distance between the spheres 𝑥 <<. Find the rate 𝑑𝑞/𝑑𝑡 with which
the charge leaks off
each sphere if their approach velocity varies as 𝑣 = 𝑎/√𝑥, where 𝑎 is a constant.

4
(Physics)

ANSWER KEY
Q.1 5400 N, attractive.
(4î−3k̂)
Q.2 |F| = 0.18 N, F̂ = .
5
q q q
Q.3 q 0 K [ r21 + r22 … … . r220 ] = 1.89 × 106 N
1 2 20

Kq1 q2
Q.4 (i) = 0.144 N
r2
Kq1 q2
(ii) = 6 m/s 2
mr2

(iii) No (Magnitude is same but direction is different)


Q.5 ±1.0 × 10−6 C, ∓3 × 10−6 C
Q.6 tan−1 (1/100) = 0.6∘
2ℓ
Q.7 from charge 4e ( If q is positive stable, If q is negative unstable)
3

Q.8 (a) moves towards the centre


q
(b) −
√3
d 4Qq
Q.9 , 2
2√2 3√3πε 0d

Q.10 (D)
Q.11 (A)
Q.12 (A)
Q.13 (C)
Q.14 (D)

Q.15 (B)

Q.16 (A)

Q.17 (D)

Q.18 (B)

Q19. (D)

Q20. (D)
Q 21. (B)
Q 22. (A)
dq 3a
Q23. = √2πε0 mg/l
dt 2

5
(Physics) ELECTROSTATICS
DPP – 2
ELECTRIC FIELD
Q.1 The electric force experienced by a charge of 5 × 10−6 C is 25 × 10−3 N. Find the magnitude of
the electric field at that position of the charge due to the source charges.
Q.2 A uniform electric field E = 91 × 10−6 V/m is created between two parallel, charged plates as
shown in figure. An electron enters the field symmetrically between the plates with a speed v0 =
4 × 103 m/s. The length of each plate is ℓ = 1 m. Find the angle of deviation of the path of the
electron as it comes out of the field. (Mass of the electron is m = 9.1 × 10−31 kg and its charge is
e = −1.6 × 10−19 C ).

Q.3 Two point particles A and B having charges of 4 × 10−6 C and −64 × 10−6 C respectively are held
at a separation of 90 cm. Locate the point(s) on the line AB or on its extension where the electric
field is zero
Q.4 Three point charges q 0 are placed at three corners of square of side a. Find out electric field
intensity at the fourth corner.
Q.5 Two point charges 3μC and 2.5μC are placed at point A(1,1,2)m and B(0,3, −1)m respectively.
Find out electric field intensity at point C(3,3,3)m.
Q.6 A hollow sphere of radius a carries a total charge Q distributed uniformly over its surface. A small
area dA of the sphere is cut off. Find the electric field at the centre due to the remaining sphere.
Q.7 (i) Two infinitely long line charges each of linear charge density λ are placed at an angle θ as
shown in figure. Find out electric field intensity at a point P, which is at a distance x from point
O along angle bisector of line charges.
(ii) Repeat the above question if the line charge densities are λ and −λ. as shown in figure.

1
(Physics) ELECTROSTATICS
Q.8 The bob of a simple pendulum has a mass of 60 g and a positive charge of 6 × 10−6 C. It makes 30
oscillations in 50 s above earth's surface. A vertical electric field pointing upward and of
magnitude 5 × 104 N/C is switched on. How much time will it now take to complete 60
oscillations?
(g = 10 m/s 2 )
Q.9 If three infinite charged sheets of uniform surface charge densities σ, 2σ and −4σ are placed as
shown in figure. Then find out electric field intensities at points A, B, C and D.

Q.10 Find out electric field intensity due to uniformly charged solid non-conducting sphere of volume
charge density ρ and radius R at following points :
(i) At a distance r from surface of sphere (inside)
(ii) At a distance r from the surface of sphere (outside)
Q.11 Repeat the question if sphere is a hollow non-conducting sphere of radius R and has uniform
surface charge density σ.
Q.12 A thread carrying a uniform charge λ per unit length has the configuration shown in figure a and
b. Assuming a curvature radius r to be considerably less than the length of the thread, find the
magnitude of the electric field strength at the point O.

Q.13 A simple pendulum has a length ℓ & mass of bob m. The bob is given a charge q coulomb. The
pendulum is suspended in a uniform horizontal electric field of strength E as shown in figure,
then calculate the time period of oscillation when the bob is slightly displaced from its mean
position.

2
(Physics) ELECTROSTATICS

ℓ ℓ ℓ ℓ
(A) 2π√g (B) 2π√{ qE } (C) 2π√{ qE } (D) 2π√ 2
g+ g− √g2 +(qE)
m m
m

Q.14 Charges 2Q and −Q are placed as shown in figure. The point at which electric field intensity is
zero will be:

(A) Somewhere between −Q and 2Q


(B) Somewhere on the left of −Q
(C) Somewhere on the right of 2Q
(D) Somewhere on the perpendicular bisector of line joining −Q and 2Q
Q.15 The maximum electric field intensity on the axis of a uniformly charged ring of charge q and
radius R will be:
1 q 1 2q 1 2q 1 3q
(A) 4πε 2
(B) 4πε 2
(C) 4πε 2
(D) 4πε 2
0 3√3R 0 3R 0 3√3R 0 2√3R

Q.16 A charged particle of charge q and mass m is released from rest in a uniform electric field E.
Neglecting the effect of gravity, the kinetic energy of the charged particle after time ' t ' seconds
is
Eqm E2 q2 t2
(A) (B)
t 2m
2E2 t2 Eq2 m
(C) (D)
mq 2t2

Q.17 A flat circular fixed disc has a charge +Q uniformly distributed on the disc. A charge +q is thrown
with kinetic energy K, towards the disc along its axis. The charge q :
(A) may hit the disc at the centre
(B) may return back along its path after touching the disc
(C) may return back along its path without touching the disc
(D) any of the above three situations is possible depending on the magnitude of K
Q.18 The linear charge density on upper half of a segment of ring is λ and at lower half, it is −λ. The
direction of electric field at centre O of ring is :

(A) along OA (B) along OB


(C) along OC (D) along OD

3
(Physics) ELECTROSTATICS
Q.19 A positively charged pendulum is oscillating in a uniform electric field as shown in Figure. Its
time period of SHM as compared to that when it was uncharged.

(mg > qE)


(A) Will increase (B) Will decrease
(C) Will not change (D) Will first increase then decrease
Q.20 There is a uniform electric field in X-direction. If the work done by external agent in moving a

charge of 0.2 C through a distance of 2 metre slowly along the line making an angle of 60∘ with
X-direction is 4 joule, then the magnitude of E is:

(A) √3 N/C (B) 4 N/C (C) 5 N/C (D) 20 N/C

Q.21 Under the influence of the Coulomb field of charge +Q, a charge −q is moving around it in an
elliptical orbit. Find out the correct statement(s). [JEE 2009, 4/160, -1]
(A) The angular momentum of the charge −q is constant.

(B) The linear momentum of the charge −q is constant.


(C) The angular velocity of the charge −q is constant.
(D) The linear speed of the charge −q is constant.

Q.22 A solid sphere of radius R has a charge Q distributed in its volume with a charge density
ρ = kr a , where k and a are constants and r is the distance from its centre. If the electric field at
r = R/2 is 1/8 times that at r = R, find the value of a. [JEE 2009, 4/160, -1]
Q.23 A uniformly charged thin spherical shell of radius R carries uniform surface charge density of σ

per unit area. It is made of two hemispherical shells, held together by pressing them with force
F (see figure). F is proportional to [JEE 2010, 5/163, -2]

1 1 1 σ2 1 σ2
(A) ε σ2 R2 (B) ε σ2 R (C) ε (D) ε
0 0 0 R 0 R2

Q.24 A wooden block performs SHM on a frictionless surface with frequency, v0 . The block carries a
charge +Q on its surface. If now a uniform electric field E is switched-on as shown, then the SHM
of the block will be [JEE 2011, 3/160, -1]

4
(Physics) ELECTROSTATICS

(A) of the same frequency and with shifted mean position.


(B) of the same frequency and with the same mean position.
(C) of changed frequency and with shifted mean position.
(D) of changed frequency and with the same mean position.
Q.25 The figures below depict two situations in which two infinitely long static line charges of
constant positive line charge density λ are kept parallel to each other. In their resulting electric
field, point charges q and −q are kept in equilibrium between them. The point charges are
confined to move in the x-direction only. If they are given a small displacement about their
equilibrium positions, then the correct statement (s) is (are):
[JEE (Advanced) 𝟐𝟎𝟏𝟓; 𝐏 − 𝟏 4/88, -2]

(A) Both charges execute simple harmonic motion.


(B) Both charges will continue moving in the direction of their displacement.
(C) Charge +q executes simple haramonic motion while charge −q continues moving in the
direction of its displacement.
(D) Charge −q executes simple haramonic motion while charge +q continues moving in the
direction of its displacement.
Q
Q.26 Let ρ(r) = πR4 r be the charge density distribution for a solid sphere of radius R and total

charge Q. For a point ' P ' inside the sphere at distance r1 from the centre of sphere, the
magnitude of electric field is : [AIEEE 2009, 4/144]
Q Qr21 Qr21
(A) 4πε 2 (B) 4πε 4
(C) 3πε 4
(D) 0
0 r1 0R 0R

Paragraph for Question Nos. 27 to 29


The nuclear charge (Ze) is non-uniformly distributed within a nucleus of radius R. The charge
density ρ(r) [charge per unit volume] is dependent only on the radial distance r from the
centre of the nucleus as shown in figure. The electric field is only along the radial direction.
[JEE 𝟐𝟎𝟎𝟖; 𝟒 × 𝟑 = 𝟏𝟐/𝟏𝟔𝟑]

5
(Physics) ELECTROSTATICS

Q.27 The electric field at r = R is :


(A) independent of a (B) directly proportional to a
(C) directly proportional to a2 (D) inversely proportional to a
Q.28 For a = 0, the value d (maximum value of ρ as shown in the figure) is:
3Ze2 3Ze 4Ze Ze
(A) 4πR3 (B) πR3 (C) 3πR3 (D) 3πR3

Q.29 The electric field within the nucleus is generally observed to be linearly dependent on r.
This implies
(A) a = 0 (B) a = R/2 (C) a = R (D) a = 2R/3

6
(Physics) ELECTROSTATICS
𝐹
Q.1 = 5 × 103 𝑁/𝐶
𝑞
𝑒𝐸ℓ
Q.2 The electron deviates by an angle 𝜃 = 𝑡𝑎𝑛−1 ( from x axis ) = 45∘
𝑚𝑣02

Q.3 30 cm from A along BA


1 Kq0
Q.4 (√2 + 2) a2

Q.5 2540î + 2000ĵ + 1720k̂N/C


QdA
Q.6 16π2 ε0 a4
4 Kλ
Q.7 (i) ; along OP.
x
4 Kλ θ
(ii) cot 2; Perpendicular to OP.
x

Q.8 100√2 ≈ 141 s


Q.9 ⃗EA = −σ ĵ, ⃗EB = −3σ ĵ,
2ε 0 2ε 0

−7σ σ
⃗EC = ĵ, ⃗ED = 2ε ĵ,
2ε0 0

ρ(R−r)
Q.10 (i) 3ε0

ρR3
(ii) 3ε 2
0 (r+R)

Q.11 (i) 0
σR2
(ii) ε 2

0 (r+R)

√2λ
Q.12 (a) E = 4πϵ
0r

(b) E = 0
Q.13 (D)
Q.14 (B)
Q.15 (C)
Q.16 (B)
Q.17 (D)
Q.18 (C)
Q.19 (A)
Q.20 (D)
Q.21 (A)
Q.22 (2)
Q.23 (A)
Q.24 (A)

7
(Physics) ELECTROSTATICS
Q.25 (C)
Q.26 (B)
Q.27 (A)
Q.28 (B)
Q.29 (C)

8
(Physics) ELECTROSTATICS

DPP-3
Electric Lines of Force, Flux Calculation and Gauss's Law
Q.1 ⃗ = 2î −
Find out the electric flux through an area 10 m2 Iying in XY plane due to an electric field E
10ĵ + 5k̂.
Q.2 In a uniform electric field E if we consider an imaginary cubical closed surface of side a, then
find the net flux through the cube?
Q.3 Find the flux of the electric field through a spherical surface of radius R due to a charge of
8.85 × 10−8 C at the centre and another equal charge at a point 3R away from the centre (Given :
ε0 = 8.85 × 10−12 units)

Q.4 A charge q is placed at the centre of an imaginary hemispherical surface. Using symmetry
arguments and the Gauss's law, find the electric flux due to this charge through the given surface.

Q.5 What do you predict by the given statement about the nature of charge (positive or negative)
enclosed by the closed surface. "In a closed surface, lines which are leaving the surface are double
than the lines which are entering it".
Q.6 A square of side 'a' is lying in xy plane such that two of its sides are lying on the axis. If an electric
⃗ = E0 xk̂ is applied on the square. The flux passing through the square is :-
field E
E0 a3 E0 a3 E0 a2
(A) E0 a3 (B) (C) (D)
2 3 2

Q.7 If electric field is uniform, then the electric lines of forces are:
(A) Divergent (B) Convergent
(C) Circular (D) Parallel
Q.8 The figure shows the electric lines of force emerging from a charged body. If the electric fields at
A and B are EA and EB respectively and if the distance between A and B is r, then

1
(Physics) ELECTROSTATICS
EB EB
(A) EA < EB (B) EA > EB (C) EA = (D) EA =
r r2

Q.9 Select the correct statement :


(A) The electric lines of force are always closed curves
(B) Electric lines of force are parallel to equipotential surface
(C) Electric lines of force are perpendicular to equipotential surface
(D) Electric line of force is always the path of a positively charged particle.
Q.10 If the electric flux entering and leaving a closed surface are respectively of magnitude ϕ1 and ϕ2 ,
then the electric charge inside the surface will be:
ϕ2 −ϕ1
(A) (B) (ϕ1 − ϕ2 )ε0
ε0

(C) ε0 (ϕ2 − ϕ1 ) (D) ε0 (ϕ2 + ϕ1 )


Q.11 An electric dipole is placed at the centre of a sphere. Mark the correct options.
(A) The electric field is zero at every point of the sphere.
(B) The flux of the electric field through the sphere is non-zero.
(C) The electric field is zero on a circle on the sphere.
(D) The electic field is not zero anywhere on the sphere.
Q.12 Figure (i) shows an imaginary cube of edge length L. A uniformly charged rod of length 2 L moves
towards left at a small but constant speed v. At t = 0, the left end of the rod just touches the
centre of the face of the cube opposite to it. Which of the graphs shown in fig.(ii) represents the
flux of the electric field through the cube as the rod goes through it ?

(A) a (B) b (C) c (D) d


Q.13 Electric charges are distributed in a small volume. The flux of the electric field through a
spherical surface of radius 20 cm surrounding the total charge is 50 V − m. The flux over a
concentric sphere of radius 40 cm will be:
(A) 50 V − m (B) 75 V − m
(C) 100 V − m (D) 200 V − m
Q.14 Eight point charges (can be assumed as uniformly charged small spheres and
their centres at the corner of the cube) having value q each are fixed at
vertices of a cube. The electric flux through square surface ABCD of the cube
is
q q q q
(A) (B) (C) (D)
24ϵ0 12ϵ0 6∈0 8ϵ0
2
(Physics) ELECTROSTATICS

Q.15 Figure shows two large cylindrical shells having uniform linear charge densities +λ and −λ.
Radius of inner cylinder is ' a ' and that of outer cylinder is ' b '. A charged particle of mass m,
charge q revolves in a circle of radius r. Then, its speed ' v ' is : (Neglect gravity and assume the
radii of both the cylinders to be very small in comparison to their length.)
λq 2λq
(A) √2πϵ (B)√πϵ
0m 0m

λq λq
(C) √πϵ (D) √4πε
0m 0m

Q.16 A charge Q is placed at a distance of 4R above the centre of a disc of radius


R. The magnitude of flux through the disc is ϕ. Now a hemispherical shell
of radius R is placed over the disc such that it forms a closed surface. The
flux through the curved surface (taking direction of area vector along
outward normal as positive), is –
(A) zero (B) ϕ
(C) −ϕ (D) 2ϕ
Q.17 The given figure gives electric lines of force due to two charges q1 and q 2 . What are the signs of
the two charges?
(A) Both are negative
(B) Both are positive
(C) q1 is positive but q 2 is negative
(D) q1 is negative but q 2 is positive
Q.18 Three positive charges of equal value q are placed at the vertices of an equilateral triangle. The
resulting lines of force should be sketched as in : [JEE 2001(Scr.), 3/105]

(A) (B) (C) (D)

Q.19 In the given figure flux through surface S1 is ϕ1 & through S2 is ϕ2 . Which is correct?

(A) ϕ1 = ϕ2 (B) ϕ1 > ϕ2 (C) ϕ1 < ϕ2 (D) None of these

3
(Physics) ELECTROSTATICS

Q.20 Statement 1: A charge is outside the Gaussian sphere of radius R. Then electric field on the
surface of sphere is zero. And
qin
⃗ ⋅ ds =
Statement 2 : As ∮ E ⃗ ⋅ ds = 0.
, for the sphere q in is zero, so ∮ E
ε0

(A) Statement-1 is true, statement-2 is true and statement-2 is correct


explanation for statement-1.
(B) Statement-1 is true, statement-2 is true and statement-2 is NOT the correct explanation for
statement-1.
(C) Statement-1 is true, statement-2 is false.
(D) Statement-1 is false, statement- 2 is true.
Q.21 A line of charge extends along a X-axis whose linear charge density varies
directly as x. Imagine a spherical volume with its centre located on X-axis
and is moving gradually along it. Which of the graphs shown in figure
correspond to the flux ϕ with the x coordinate of the centre of the volume?
(A) a (B) b (C) c (D) d
Q.22 Column II corresponds to the graph of magnitude of electric field versus distance from centre of
charge distribution in Column I.
Column-I Column-II

(A) Ring along its axis (P)

(B) Uniformly charged solid sphere (Q)

(C) Uniformly charged spherical shell (R)

(D) Combination of charge +Q and - Q at (S)


the perpendicular bisector

(T)

4
(Physics) ELECTROSTATICS

Q.23 A point charge +Q is positioned at the center of the base of a square pyramid as shown. The flux
through one of the four identical upper faces of the pyramid is :-

Q Q Q
(A) (B) (C) (D) None of these
16ε0 4ε0 8ε0

Q.24 An infinite, uniformly charged sheet with surface charge density 𝜎 cuts through a spherical
Gaussian surface of radius R at a distance x from its center, as shown in the figure. The electric
flux Φ through the Gaussian surface is :-

πR2 σ 2π(R2 −x2 )σ π(R−x)2 σ π(R2 −x2 )σ


(A) (B) (C) (D)
ε0 ε0 ε0 ε0

Q.25 A point charge q is placed at a point inside a hollow conducting sphere. Which of the following
electric force pattern is correct? [IIT-JEE'2003 (scr)]

(A) (B)

(C) (D)

5
(Physics) ELECTROSTATICS

ANSWER KEY
N−m2 q
1. 50Nm2 /C. 2. 0 3. 104 ,0 4.
C 2ε0

5. There is net positive charge in the close surface.


6. (B) 7. (D) 8. (B) 9. (C)
10. (C) 11. (D) 12. (D) 13. (A)
14. (C) 15. (A) 16. (C) 17. (A)
18. (B) 19. (A) 20. (D) 21. (C)
22. (A) → (Q); (B) → (S); (C) → (R); (D) → (P)
23. (C) 24. (D) 25. (A)

6
(Physics) ELECTROSTATICS
DPP - 4
APPLICATION OF GAUSS’S THEOREM & CONDUCTOR
Q.1 A solid metallic sphere is place in a uniform electric field. Which
of the curves shown in the figure represent the lines of force
correctly?
(A) (a) (B) (b) (C) (c) (D) (d)
Comprehension
Figure shows three concentric spherical conductors A, B and C with radii
R, 2R and 4R respectively. A and C are connected by a conducting wire and B
is having a uniform charge +Q. Based on the above facts, answer the
following questions.
Q.2 Charge on the conductor A is qA and that on C is qC . Then
Q Q 2Q
(A) qA = qc = − (B) qA = and qC =
3 3 3
Q Q Q Q
(C) qA = and qC = − (D) qA = − and qC =
3 3 3 3

Q.3 The potential of A is


Q Q Q
(A) − (B) − (C) (D) None of these
4πε0 R 12πε0 R 16πε0 R

Q.4 The potential of B is


Q 5Q 3Q 5Q
(A) (B) (C) (D)
4πε0 R 12πε0 R 6πε0 R 48πε0 R

Comprehension
Two spherical cavities, of radii a and b are hollowed out from the interior
of a neutral conducting sphere of radius R. At the centre of each cavity, a
point charge is placed. Call these charges qa and qb , distance between qa
and qb is r. Based on the above facts, answer the following questions.
Q.5 The force on charge qa is.
qa qb qa (qa +qb ) (qa )(qa +qb )
(A) (B) (C) (D) zero
4πε0 r2 4πε0 R2 4πε0 r2

Q.6 The field outside the conductor at any point P at a distance r0 (> R) from centre of conductor?
qa qa +qb qa +qb qb
(A) (B) (C) (D)
4πε0 r20 4πε0 R2 4πε0 r20 4πε0 r20

Q.7 In each cavity, there is certain electric field say E. If another charge qc were brought near the
conductor then E
(A) increases. (B) decreases.
(C) remains same. (D) depends on the nature of charge qc .

1
(Physics) ELECTROSTATICS
Comprehension
A ball of radius R carries a positive charge whose volume charge density depends only on the
distance r form the ball's centre as.
r
ρ = ρ0 (1 − )
R
where ρ0 is a constant. Assume ε as the permittivity of the ball, answer the following questions.
Q.8 The magnitude of the electric field as a function of the distance r outside the ball is given by
ρ0 R3 ρ0 R3 ρ0 R2 ρ0 R2
(A) E = (B) E = (C) E = (D) E =
8εr2 12εr2 8εr3 12εr3

Q.9 The value of distance rm , at which electric field intensity is maximum, is given by
R 3R 2R 4R
(A) rm = (B) rm = (C) rm = (D) rm =
3 2 3 3

Q.10 The maximum electric field intensity is


ρ0 R ρ0 ε ρ0 R ρ0 R
(A) Em = (B) Em = (C) Em = (D) Em =
9ε 9R 3ε 6ε

Q.11 In figure a conducting sphericai shell of inner radius x and outer radius y
is concentric with a larger conducting spherical shell of inner radius a
and outer radius b. The inner shell has a total charge +3Q and the outer
shell has a total charge +5Q. Consider a point P arbitrarily taken in the
regions of the system of these two shells. Based on the facts provided,
match the contents of COLUMN-I with the locations of the point P
mentioned in COLUMN-II.

COLUMN-I COLUMN-II

(A) Electric field strength is (p) Outer surface of the larger spherical
zero shell
(B) Electric field strength is (q) Inner surface of the larger spherical shell
non-zero
(C) Magnitude of charge on this (r) Outer surface of the smaller spherical
surface is 3Q shell
(D) Charge on this surface is (s) Region between inner and outer surface
+8Q of bigger shell
(t) Region between inner and outer surface
of smaller shell

2
(Physics) ELECTROSTATICS
ANSWER KEY

1. (D) 2. (D) 3. (C) 4. (D) 5. (D) 6. (C) 7. (C)

8. (B) 9. (C) 10. (A) 11. A→ (s, t); B→(p ,q, r); C→(q, r); D→(p)

3
(Physics) ELECTROSTATICS
DPP - 5
Potential & Potential Energy
Q.1 Two charges, +q and −q, each of mass m, are revolving in a circle of radius R, under mutual
electrostatic force. Find (a) speed of each charge (b) kinetic energy of system (c) potential energy
of the system (d) total energy of the system
Q.2 Calculate the work required to be done to make an arrangement of three particles each having a
charge +q such that the particles lie at the vertices of an equilateral triangle of side a. What work
will be done by electric field when the particles are shifted away so that the side of triangle
becomes 2a ?
Q.3 Determine the interaction energy of the point charges located at the corners of a square of side
ℓ in the figures shown.

Q.4 Two charges 6μC and −4μC are located 15 cm apart. At what point on the line joining the two
charges is the electric potential zero? Take the potential at infinity to be zero.
Q.5 Consider a system of two charges shown in figure. The arrangement is also called an Electric
Dipole. Find the electric potential at an arbitrary point on the x-axis and make a plot of
V(x) x q
Vs whereV0 = .
V0 a 4πε0 a

Electric dipole
Q.6 On a thin rod of length ℓ = 1 m, lying along the x-axis with one end at the origin x = 0, there is
uniformly distributed charge per unit length λ = Kx, where K = 10−9 Cm−2 . Find the work done
in displacing a charge q = 1000μC from a point A(0, √0.44)m to B(0, ℓ)m.
Q.7 There are two thin wire rings, each of radius R, whose axes coincide. The charges of the rings are
q and −q. Find the potential difference between the centres of the rings separated by a distance
a. Also calculate the work done to move a charge q 0 from centre of first ring to the centre of other
ring.
1
(Physics) ELECTROSTATICS
Q.8 A thin spherical conducting shell of radius R has a charge q. Another charge Q is placed at the
R
centre of the shell. The electrostatic potential at a point P at a distance 2 from the centre of the

shell is :
2Q 2Q 2q
(A) 4πϵ (B) 4πϵ − 4πϵ
0R 0R 0R

2Q q (q+Q) 2
(C) 4πϵ + 4πϵ (D)
0R 0R 4πϵ0 R

Q.9 A charged particle q is shot towards another charged particle Q which is fixed, with a speed v. It
approaches Q upto a closest distance r and then returns. If q was given a speed 2v, the closest
distance of approach would be :

(A) r (B) 2r (C) r/2 (D) r/4


Q.10 Two thin wire rings each having a radius R are placed at a distance d apart with their axes
coinciding. The charges on the two rings are +q and −q. The potential difference between the
centres of the two rings is :
qR q 1 1 q 1 1
(A) 4π∈ 2
(B) 2πϵ [R − √R2 ] (C) Zero (D) 4πϵ [R − √R2 ]
0d 0 +d2 0 +d2

Q.11 Two spherical conductors A and B of radii 1 mm and 2 mm are separated by a distance of 5 cm
and are uniformly charged. If the spheres are connected by a conducting wire then in equilibrium
condition, the ratio of the magnitude of the electric fields at the surfaces of spheres A and B is :
(A) 4: 1 (B) 1: 2 (C) 2: 1 (D) 1: 4
Q.12 Charges are placed on the vertices of a square as shown. Let ⃗E be the electric
field and V the potential at the centre. If the charges on A and B are
interchanged with those on D and C respectively, then :
⃗ remains unchanged, V changes
(A) E ⃗ and V change
(B) both E
(C) ⃗E and V remain unchanged (D) ⃗E changes and V remains unchanged
Q.13 An electric charge 10−3 μC is placed at the origin (0,0) of X − Y coordinate system. Two points A
and B are situated at (√2, √2) and (2,0) respectively. The potential difference between the points
A and B will be :
(A) 9 V (B) Zero (C) 2 V (D) 4.5 V
Q.14 Two points P and Q are maintained at the potential of 10 V and −4 V, respectively. The work done
in moving 100 electrons from P to Q is :
(A) −2.24 × 10−16 J (B) 2.24 × 10−16 J
(C) −9.60 × 10−17 J (D) 9.60 × 10−17 J

2
(Physics) ELECTROSTATICS
Q.15 A charge of total amount Q is distributed over two concentric hollow spheres of radii r and
R(R > r) such that the surface charge densities on the two spheres are equal. The electric
potential at the common centre is :
1 (R−r)Q 1 (R+r)Q 1 (R+r)Q 1 (R−r)Q
(A) 4πε (R 2 +r2 )
(B) 4πε 2 2
(C) 4πε 2 +r2 )
(D) 4πε 2 +r2 )
0 0 2(R +r ) 0 (R 0 (R

Q.16 A charge Q is uniformly distributed over a long rod AB of length L as shown in the figure. The
electric potential at the point O lying at a distance L from the end A is :

Q 3Q Q Qln2
(A) 8πϵ (B) 4πϵ (C) 4π∈ (D) 4πϵ
0L 0L 0 Lln2 0L

Q.17 The potential (in volts) of a charge distribution is given by


V(z) = 30 − 5z 2 for |z| ≤ 1 m
V(z) = 35 − 10|z| for |z| ≥ 1 m
V(z) does not depend on x and y. If this potential is generated by a constant charge per unit
volume ρ0 (in units of ε0 ) which is spread over a certain region, then choose the correct
statement:
(A) ρ0 = 20ε0 in the entire region
(B) ρ0 = 10ε0 for |z| ≤ 1 m and p0 = 0 elsewhere
(C) ρ0 = 20ε0 for |z| ≤ 1 m and p0 = 0 elsewhere
(D) ρ0 = 40ε0 in the entire region
Q.18 Within a spherical charge distribution of charge density ρ(r), N equipotential surfaces of
potential V0 , V0 + ΔV, V0 + 2ΔV, … V0 + NΔV(ΔV > 0), are drawn and have increasing radii r0 , r1 ,
r2 , … rN , respectively. If the difference in the radii of the surfaceg is constant for all values of V0
and ΔV then :
1
(A) ρ(r) = constant (B) ρ(r) ∝ r2
1
(C) ρ(r) ∝ (D) ρ(r) ∝ r
r

Q.19 Three concentric metal shells A, B and C of respective radii a, b and c(a < b < c) have surface
charge densities +σ, −σ and +σ respectively. The potential of shell B is :
σ a2 −b2
(A) ϵ [ + c]
0 a

σ a2 −b2
(B) ϵ [ + c]
0 b

σ b2 −c2
(C) ϵ [ + a]
0 b

σ b2 −c2
(D) ϵ [ + a]
0 c

3
(Physics) ELECTROSTATICS
Q.20 A positive point charge is released from rest at a distance r0 from a
positive line charge with uniform density. The speed (v) of the point
charge, as a function of instantaneous distance r from line charge, is
proportional to :
r
(A) v ∝ e+r/r0 (B) v ∝ ln⁡ (r )
0

r r
(C) v ∝ (r ) (D) v ∝ √ln⁡ (r )
0 0

Q.21 Statement-1: For practical purposes, the earth is used as a reference at zero potential in electrical
circuits.
Statement-2: The electrical potential of a sphere of radius R with charge Q uniformly distributed
Q
on the surface is given by 4πz R.
0

(A) If both statements are TRUE and STATEMENT 2 is the correct explanation of STATEMENT 1.
(B) If both statements are TRUE but STATEMENT 2 is not the correct explanation of STATEMENT 1.
(C) If STATEMENT 1 is TRUE and STATEMENT 2 is FALSE.
(D) If STATEMENT 1 is FALSE but STATEMENT 2 is TRUE.
Q.22 Statement-1: Induced charge does not contribute to electric field or potential at a given point.
Statement-2: A point change q 0 is kept outside a solid metallic sphere, the electric field inside
the sphere is zero.
(A) If both statements are TRUE and STATEMENT 2 is the correct explanation of STATEMENT 1.
(B) If both statements are TRUE but STATEMENT 2 is not the correct explanation of STATEMENT 1.
(C) If STATEMENT 1 is TRUE and STATEMENT 2 is FALSE.
(D) If STATEMENT 1 is FALSE but STATEMENT 2 is TRUE.
Comprehension 23 to 25
Two positive point charges each of magnitude q are placed on the y-axis at the points (0, a) and
(0, −a). A positively charged particle of charge 𝑞0 and mass 𝑚, when displaced slightly from the
origin in the direction of the negative 𝑥-axis attains a speed 𝑣0 at infinity. When the particle is
projected towards the left along the 𝑥-axis from a point at a large distance towards the right of
the origin with a velocity half the value of 𝑣0 , then it comes to rest at a point 𝑃(𝑥, 0). Again, if a
negatively charged particle from the rest (assume the negatively charged particle to be at very
large distance towards the left of origin), then this negatively charged particle crosses the origin
with a speed 𝑣. Based on the above facts, answer the following questions.

4
(Physics) ELECTROSTATICS
Q.23 The value of v0 is
qq0 qq0
(A) √2πε (B) √πε
0 ma 0 ma

qq0
(C) √3πε (D) None of these
0 ma

Q.24 The value of x is


(A) √3a (B) 4a (C) √2a (D) √15a
Q.25 The value of v is
v0 v0 v0
(A) (B) (C) v0 (D)
3 2 4

5
(Physics) ELECTROSTATICS
ANSWER KEY
𝑞2 q2 q2 q2
1. (a) √ (b) (c) − (d) −
16𝜋𝜀0 𝑅 16πε0 R 8πε0 R 16πε0 R

3q2 q2 q2 √2q2
2. 3. (a) 4πε l (4 + √2) (b) 4πε l (−4 + √2) (c) − (4πε l)
8πε0 a 0 0 0

4. (a) by between the charge x = 5 cm from 6μC


(b) outside the charge x = 45 cm from 6μC

qq 1 1
5. 6. WA→B = −1.1 × 10−3 J 7. WA→B = 2πε0 [√R2 − R]
0 +a2

8. (C) 9. (D) 10. (B) 11. (C) 12. (D) 13. (B) 14. (B)
15. (C) 16. (D) 17. (B) 18. (C) 19. (B) 20. (C) 21. (B)
22. (D) 23. (B) 24. (D) 25. (C)

6
(Physics) ELECTROSTATICS
DPP-6
⃗ AND V:
QUESTIONS BASED ON RELATION BETWEEN 𝐄
Q.1 ⃗ = 2yî + 2xĵ, then find V(x, y, z)
If E
Q.2 If V = x 2 y + y 2 z then find ⃗E(x, y, z)
Q.3 If V = 2r 2 then find out (i) ⃗E(1,0, −2) (ii) ⃗E(r = 2)
Q.4 An electric field ⃗E = (10î + 20ĵ)N/C exists in the space. If the potential at the origin is taken to
be zero, find the potential at (3 m, 3 m).
Q.5 An electric field ⃗E = Bxî exists in space, where B = 20 V/m2 . Taking the potential at (2 m, 4 m)
to be zero, find the potential at the origin.
Q.6 If E = 2r 2 , then find V(r)
Q.7 If ⃗E = 2x 2 î − 3y 2 ĵ, then find V(x, y, z)
Q.8 The variation of potential with distance r from a fixed point is shown in Figure. The electric field
at r = 5 cm, is :
(A) (2.5)V/cm
(B) (−2.5)V/cm
(C) (−2/5)cm
(D) (2/5)V/cm
Q.9 In the above question, the electric force acting on a point charge of 2C placed at the origin will be :
(A) 2 N (B) 500 N
(C) −5 N (D) −500 N
Q.10 The electric potential V as a function of distance x (in metre) is given by V = (5x 2 + 10x − 9)
volt.
The value of electric field at x = 1 m would be :
(A) −20volt/m (B) 6volt/m
(C) 11volt/m (D) −23volt/m
Q.11 A uniform electric field having a magnitude E0 and direction along positive x-axis exists. If the
electric potential V is zero at x = 0, then its value at x = +x will be :
(A) Vx = xE0 (B) Vx = −xE0
(C) Vx = x 2 E0 (D) Vx = −x 2 E0
Q.12 Let E be the electric field and V, the electric potential at a point.
(A) If E ≠ 0, V cannot be zero
(B) If E = 0, V must be zero
(C) If V = 0, E must be zero

1
(Physics) ELECTROSTATICS
(D) None of these
Q.13 The electric field in a region is directed outward and is proportional to the distance r from the
origin. Taking the electric potential at the origin to be zero, the electric potential at a distance r :
(A) increases as one goes away from the origin.
(B) is proportional to r 2
(C) is proportional to r
(D) is uniform in the region
Q.14 A non-conducting ring of radius 0.5 m carries a total charge of 1.11 × 10−10 C distributed non-
uniformly on its circumference producing an electric field ⃗E every where in space. The value of
ℓ=0
⃗ (ℓ = 0 being centre of the ring) in volts is :
⃗ ⋅ dℓ
the line integral ∫ℓ=∞ − E
Approximately)
(A) +2 (B) -1
(C) -2 (D) zero

2
(Physics) ELECTROSTATICS
ANSWER KEY
Q.1 −2xy + C
Q.2 −2xyî − (x 2 + 2yz)ĵ − y 2 k̂
Q.3 (i)−4(î − 2k̂)
⃗ = −8r̂
(ii) E
Q.4 V(3,3) − V(0,0) = −90 V
Q.5 40 V
−2r3
Q.6 +C
3
2x3
Q.7 − + y3 + C
3

Q.8 (A)
Q.9 (D)
Q.10 (A)
Q.11 (B)
Q.12 (D)
Q.13 (B)
Q.14 (A)

3
(Physics) ELECTROSTATICS
DPP-7
Dipole
Q.1 An electric dipole is placed making at an angle 60∘ with an electric field of strength 4 × 105 N/C.
It experiences a torque equal to 8√3 N − m. Calculate the charge on the dipole, if it is of length
4 cm.
Q.2 A system has two charges q A = 2.5 × 10−7 C and q B = −2.5 × 10−7 C located at points A ≡
(0,0, −15)cm and B ≡ (0,0,15)cm respectively. What are the total charge and electric dipole
moment vector of the system?
Q.3 An electric dipole has the magnitude of its charge as q and its dipole moment is p. It is placed in
a uniform electric field E. If its dipole moment is along the direction of the field, the force on it
and its potential energy are respectively:-
(A) q. E and p. E (B) zero and minimum
(C) q. E and maximum (D) 2q. E and minimum
Q.4 The electric potential at a point due to an electric dipole will be :
⃗ ⋅r⃗
P ⃗ ⋅r⃗
P ⃗ ×r⃗
P ⃗ ×r⃗
P
(A) k r3 (B) k r2 (C) k (D) k
r3 r2

Q.5 An electric dipole of moment p


⃗ = (−î − 3ĵ + 2k̂) × 10−29 Cm is at the origin (0,0,0). The electric
field due to this dipole at r = +î + 3ĵ + 5k̂ (note that r ⋅ p
⃗ = 0) is parallel to
(A) (+î − 3ĵ − 2k̂) (B) (−î − 3ĵ + 2k̂)
(C) (+î + 3ĵ − 2k̂) (D) (−î + 3ĵ − 2k̂)
Q.6 An electric dipole is placed on x-axis in proximity to a line charge of linear charge density
3.0 × 10−6 C/m. Line charge is placed on z-axis and positive and negative charge of dipole is at a
distance of 10 mm from the origin respectively. If total force of 4 N is exerted on the dipole, find
out the amount of positive or negative charge of the dipole
(A) 4.44μC (B) 8.8μC (C) 0.485mC (D) 815.1nC
Q.7 Two ideal electric dipoles A and B, having their dipole moment p1 and p2 respectively are placed
on a plane with their centres O as shown in the figure. At point C on the axis of dipole A, the
resultant electric field is making an angle of 37∘ with the axis. The ratio of the dipole moment of
P 3
A and B, P1 is (Take sin 37∘ = 5)
2

4 3 2 3
(A) 3 (B) 2 (C) 3 (D) 8

1
(Physics) ELECTROSTATICS
Q.8 An electric dipole is formed by two equal and opposite charges q with separation d. The charges
have same mass m. It is kept in a uniform electric field E. If it is slightly rotated from its
equilibrium orientation, then its angular frequency ω is
2qE qE qE qE
(A) √ md (B) √2md (C) 2√md (D) √md

Q.9 ⃗ is kept in a uniform electric field ⃗E then for what value of the angle
When an electric dipole p
between p ⃗ , torque will be maximum :-
⃗ and E
(A) 90∘ (B) 0∘ (C) 180∘ (D) 45∘
Q.10 What will be the ratio of electric field at the axis and at equatorial line of a dipole :-
(A) 1: 2 (B) 2: 1 (C) 4: 1 (D) 1: 4
Q.11 An electric dipole is placed in non uniform electric field, then it experiences :-
(A) no force (B) only torque
(C) only linear resultant force (D) linear resultant force and torque both
Q.12 At the mid point on the axis of an electric dipole
(A) the electric field is zero.
(B) the electric potential is zero.
(C) neither the electric field nor the electric potential is zero.
(D) the electric field is directed perpendicular to the axis of the dipole.
Q.13 The magnitude of charges of electric dipole is 3.2 × 10−19 and distance between them is 2.4Å. If
it is placed in a electric field 4 × 105 V/m then its dipole moment is (in C − m )
(A) 9.6 × 10−5 (B) 12.8 × 10−14
(C) 7.68 × 10−29 (D) 30.72 × 10−24
Q.14 Three point charges +q, −2q and +q are placed at points
(x = 0, y = a, z = 0), (x = 0, y = 0, z = 0) and (x = a, y = 0, z = 0) respectively. The magnitude
and direction of the electric dipole moment vector of this charge assembly are :-
(A) √2 qa along +x direction
(B) √2 qa 0along +y direction
(C) √2 qa along the line joining points (x = 0, y = 0, z = 0) and (x = a, y = a, z = 0)
(D) qa along the line joining points (x = 0, y = 0, z = 0) and (x = a, y = a, z = 0)
Q.15 Charges −q and +q located at A and B, respectively, constitute an electric dipole. Distance AB =
2a, O is the mid point of the dipole and OP is perpendicular to AB. A charge Q is placed at P where
OP = y and y >> 2a. The charge Q experiences an electrostatic force F. If Q is now moved along
y y
the equatorial line to P ′ such that the OP ′ = (3), force on Q will be close to (3 >> 2a)

2
(Physics) ELECTROSTATICS

F
(A) 27F (B) 3F (C) 3 (D) 9 F

Q.16 Determine the electric dipole moment of the system of three charges, placed on the vertices of
an equilateral triangle, as shown in the figure.

î+ĵ ĵ−î
(A) 2qlĵ (B) (ql) (C) √3ql (D) −√3qlĵ
√2 √2

Q.17 An electric dipole has a fixed dipole moment p


⃗ , which makes angle θ with respect to x-axis. When
subjected to an electric field ⃗E1 = Eî, it experiences a torque ⃗T1 = τk̂. When subjected to another
electric field ⃗E2 = √3E1 ĵ it experiences a torque ⃗T2 = −T
⃗ 1. The angle θ is
(A) 30∘ (B) 45∘ (C) 60∘ (D) 90∘
Q.18 Two opposite and equal charges 4 × 10−8 C when placed 2 × 10−2 cm away form a dipole. If this
dipole is placed in an external electric field 4 × 108 N/C the value of maximum torque and the
work done in rotating it through 180 degree will be :-
(A) 64 × 10−4 Nm, 64 × 10−4 J
(B) 32 × 10−4 Nm, 32 × 10−4 J
(C) 64 × 10−4 Nm, 32 × 10−4 J
(D) 32 × 10−4 Nm, 64 × 10−4 J
⃗ is lying along a uniform electric field ⃗E. The work done in
Q.19 An electric dipole of dipole moment p
rotating the dipole by 90∘ is :-
(A) 2pE
(B) pE
(C) √2pE
pE
(D) 2

3
(Physics) ELECTROSTATICS

ANSWER KEY
1. 10−3 C or 1mC 2. Total charge = 0; Dipole moment p
⃗ = −7.5 × 10−8 C − mk̂
3. (B) 4. (A) 5. (C) 6. (A) 7. (C) 8. (A) 9. (A)
10. (B) 11. (D) 12. (B) 13. (C) 14. (C) 15. (A) 16. (D)
17. (C) 18. (D) 19. (B)

4
(Physics)

EXERCISE-1
1. Four charges are arranged at the corners of a square ABCD, as shown. The force on a +ve charge
kept at the centre of the square is

(A) Zero (B) Along diagonal AC


(C) Along diagonal BD (D) Perpendicular to the side AB
2. Which of the following is a volt?
(A) Erg per cm (B) Joule per coulomb
2
(C) Erg per ampere (D) Newton/ (coulomb x m )
3. When a negative charge is released and moves in electric field, it moves toward a position of
(A) lower electric potential and lower potential energy
(B) lower electric potential and higher potential energy
(C) higher electric potential and lower potential energy
(D) higher electric potential and higher potential energy
4. Two particles X and Y, of equal mass and with unequal positive charges, are free to move and are
initially far away from each other. With Y at rest, X begins to move towards it with initial velocity
u. After a long time, finally
(A) X will stop, Y will move with velocity u
u
(B) X and Y will both move with velocities each
2
(C) X will stop, Y will move with velocity < u
u
(D) Both will move with velocities <
2
5. In a uniform electric field, the potential is 10V at the origin of coordinates, and 8V at each of the
points (1, 0, 0), (0, 1, 0) and (0, 0, 1). The potential (in volt) at the point (1, 1, 1) will be
6. Point P lies on the axis of a dipole. If the dipole is rotated by 90 anticlockwise, the electric field
vector a at P will rotate by
(A) 90 clockwise (B) 180 clockwise
(C) 90 anticlockwise (D) none of these

APNI KAKSHA 27
(Physics)

7. Electric flux through a surface of area 100m2 lying in the xy plane is (in V-m) if

E = ˆi + 2jˆ + 3kˆ
(A) 100 (B) 141.4 (C) 173.2 (D) 200
8. A hollow metal sphere of radius 5cm is charged such that the potential on its surface is 10V. The
potential (in V) at the centre of the sphere is
9. If the electric potential of the inner metal sphere is 10volt and that of the outer shell is 5 volt, then
the potential (in volt) at the centre will be

10. An isolated system consists of two charged particles of equal mass. Initially the particles are far
apart, have zero potential energy and one of the particles has non – zero speed. If the radiation is
neglected, which of the following is true for the total energy of the system at any later time?
(A) It is negative and constant
(B) It is positive and constant
(C) It is constant, but the sign cannot be determined unless the initial velocities of both particles
are known
(D) It cannot be constant of the motion because the particles exert force on each other
11. A point charge Q is located at the centre of a hollow spherical conductor of inner radius R 1 and

outer radius R 2 , the conductor being uncharged initially. The potential at the inner surface will

be

1 1  1 1 
(A) KQ  +  (B) KQ  − 
 R1 R 2   R1 R 2 
1 1
(C) KQ  −  (D) None of the above
 R 2 R1 
12. A number of spherical conductors of different radii are charged to some potential. The surface
charge density of each conductor is related with its radius as
1 1
(A)  (B)  (C) R (D) None of these
R2 R

APNI KAKSHA 28
(Physics)

13. A dipole of 2 C charges each consists of the positive charge at the point P (1, -1) and the negative
charge is placed at the point Q (-1, 1). The work done in displacing a charge of +1C from point
A ( −3, −3) to B (4, 4) is

(A) 1.6 10 3.2 10−19 J


−19
J (B) (C) zero (D) 4.8 eV

14. There are two concentric metal shells of radii r1 and r2 (  r1 ) . If the outer shell has a charge q and
the inner shell is grounded, the charge on the inner shell is

r   r2 
(A) zero (B) − 1 q (C) r1r2q (D)  q
 r2   r1 
15. If the intensity of electric field at a distance x from the centre in axial position of small electric
dipole is equal to intensity at distance y in equatorial position, then
y x x
(A) x=y (B) x= (C) y= (D) y=
2 22/3 21/3
16. Three charges Q, +q and +q are placed at the vertices of a right- angled isosceles triangle as shown
−q
in the figure. The net electrostatic energy of the configuration will be zero if Q is equal to
+ 2
. Find  +  ?

17. Three positive charges of equal value q are placed at the vertices of an equilateral triangle. The
resulting lines of force should be sketched as in

(A) (B)

(C) (D)

APNI KAKSHA 29
(Physics)

18. The variation of potential with distance R from fixed point is shown in the figure. The electric field
at R = 5m is

2  −2 
(A) 2.5 V/m (B) -2.5 V/m (C)  5 V / m (D)  5 V / m
   
19. A spherical cavity is created in a neutral solid conducting sphere. Inside the cavity, a dipole is
placed as shown in the figure. Electrostatic potential at point P only due to charge induced on the
inner surface of the cavity is (assume that  d )

q sin  −q sin  q sin  q sin 


(A) (B) (C) − (D)
40 d2 40d2 40 d 40d
20. Charge Q each are distributed uniformly on a non-conducting hemispherical shell (on the surface)
and in the volume of a non-conducting solid hemisphere. A point charge q is brought slowly at the
centre C1 of shell and at the centre C 2 of solid hemisphere from infinity.

W 
If the work done in these two processes are W1 and W2 then, the ratio  1  is
 W2 
3 3 4 2
(A) (B) (C) (D)
4 2 3 3
APNI KAKSHA 30
(Physics)

EXERCISE-2
1. Two free positive charges 4q and q are a distance apart. What charge Q is needed to achieve
equilibrium for the entire system and where should it be placed from charge q?
4 4
(A) Q = q (negative) at (B) Q = q (positive) at
9 3 9 3

(C) Q = q (positive) at (D) Q = q (negative) at


3 3
2. Six charges are placed at the corner of a regular hexagon as shown. if an electron is placed at its
centre O, force on it will be

(A) Zero (B) along OF (C) along OC (D) none of the above

3. The direction ( ) of E at point P due to uniformly charged finite rod will be

(A) at angle 30 from x – axis (B) 45 from x – axis


(C) 60 from x – axis (D) None of the above
4. Two equal negative charges are fixed at the points [0, a] and [0, -a] on the y – axis. A positive charge
Q is released from rest at the points [2a, 0] on the x – axis. The charge Q will
(A) execute simple harmonic motion about the origin
(B) move to the origin and remain at rest
(C) move to infinity
(D) execute oscillatory but not simple harmonic motion

APNI KAKSHA 31
(Physics)

5. An infinite non – conducting sheet of charge has a surface charge density of 10−7 C / m2 . The
separation between two equipotential surfaces near the sheet whose potential differ by 5V is
88
mm . Find n?
n
6. Two positively charged particles X and Y are initially far away from each other and at rest. X begins
to move towards Y with some initial velocity. The total momentum and energy of the system are p
and E
(A) If Y is fixed, both p and E are conserved
(B) If Y is fixed, E is conserved, but not p
(C) If both are free to move, p is conserved but not E
(D) If both are free, E is conserved, but not p
7. The equation of an equipotential line in an electric field is y = 2x , then the electric field strength
vector at (1, 2) may be

(A) 4iˆ + 3jˆ (B) 4iˆ + 8jˆ (C) 8iˆ + 4jˆ (D) −8iˆ + 4jˆ
8. Figure shows equi – potential surfaces for a two charges system. At which of the labelled points
will an electron have the highest potential energy?

(A) Point A (B) Point B (C) Point C (D) Point D

9. In a certain region of space, the potential is given by V = k 2x2 − y2 + z2  . The electric field at

the point (1, 1, 1) has magnitude is 2k  . Find  ?


10. An infinite uniformly charged sheet with surface charge density  cuts through a spherical
Gaussian surface of radius R at a distance x from its centre, as shown in the figure. The electric flux
 through the Gaussian surface is

APNI KAKSHA 32
(Physics)

R 2 2 ( R 2 − x 2 )  ( R − x ) 
2
( R 2 − x2 ) 
(A) (B) (C) (D)
0 0 0 0
11. A positive charge q is placed in a spherical cavity made in a positively charged sphere. The centres

of sphere and cavity are separated by a small distance . Force on charge q is

(A) in the direction parallel to vector


(B) in radial direction
(C) in a direction which depends on the magnitude of charge density in sphere
(D) direction cannot be determined

12. A non – conducting ring of radius 0.5m carries a total charge of 1.1110−10 C distributed non –
uniformly on its circumference producing an electric field E everywhere in space. The value of the
=0
line integral  −E.d
=
( = 0 being centre of the ring) in volts is

13. An ellipsoidal cavity is carved within a perfect conductor. A positive charge q is placed at the centre
of the cavity. The points A and B are on the cavity surface as shown in the figure. Then

(A) electric field near A in the cavity = electric field near B in the cavity
(B) charge density at A = charge density at B
(C) potential at A = potential at B
q
(D) total electric field flux through the surface of the cavity is
0
14. The system of charges as shown in the figure. A thick spherical shell with an inner radius a and an
outer radius b is made of conducting material. A point charge +Q is placed at the centre of the
spherical shell and a total charge –q is placed on the shell. Assume that the electrostatic potential
is zero at an infinite distance from the spherical shell. The electrostatic potential at a distance R

( a  R  b ) from the centre of the shell is

APNI KAKSHA 33
(Physics)

KQ Q−q Q−q
(A) 0 (B) (C) K (D) K
a R b
1
(where K= )
40
15. The ratio of the time periods of small oscillations of the insulated spring and mass system before
and after charging the masses is

16. A sphere of radius R is having charge Q uniformly distributed over it. The energy density of the

Q
electric field in the air, at a distance r is . Find  +  +  + P ? (r > R) is given by (in J/m3 )
 0r P

17. If Vo be the potential at origin in an electric field E = Exˆi + Eyˆj , then the potential at point P(x,
y) is
(A) Vo + xE x + yE y (B) Vo + xE x − yE y

(C) Vo − xE x − yE y (D) (x 2
+ y2 ) E2x + E2y − Vo
18. Electric charges q, q and -2q are placed at the corners of an equilateral triangle ABC of side L. The

magnitude of electric dipole moment of the system is  qL . Find  ?


19. Six charges of equal magnitude, 3 positive and 3 negative are to be placed on PQRSTU corners of a
regular hexagon, such that field at the centre is double that of what it would have been if only one
+ve charge is placed at R. identify the correct sequence of charge.

(A) +, +, +, −, −, − (B) −, +, +, +, −, − (C) −, +, +, −, +, − (D) +, −, +, −, +, −


20. A point charge q is placed at a point on the axis of a non – conducting circular plate of radius r at a

qr 
distance R (  r ) from its centre. The electric flux associated with the plate is . Find
40R
+?
APNI KAKSHA 34
(Physics)

R
21. The diagram shows part of a disc of radius R carrying uniformly distributed charge from to R of
2
density  .Electric potential at the center C is

R R R R
(A) (B) (C) (D)
2 0 16 0 24 0 32 0

APNI KAKSHA 35
(Physics)

EXERCISE-3
1. Two identical positive charges are fixed on the y – axis, at equal distances from the origin O. A
particle with a negative charge starts on the x – axis at a large distance from O, moves along the +x
– axis, passes through O and moves far away from O. Its acceleration a is taken as positive along its
direction of motion. The particle’s acceleration a is plotted against its x – coordinate. Which of the
following best represents the plot?

(A) (B)

(C) (D)
2. A non-conducting ring of radius R has uniformly distributed positive charge Q. A small part of the
ring, of length d, is removed (d<R). The electric field at the centre of the ring will now be
3
(A) directed towards the gap, inversely proportional to R
2
(B) directed towards the gap, inversely proportional to R
3
(C) directed away from the gap, inversely proportional to R
2
(D) directed away from the gap, inversely proportional to R
3. A small particle of mass m and charge –q is placed at point P on the axis of uniformly charged ring
and released. If R > x, the particle will undergo oscillations along the axis of symmetry with an

Qq
angular frequency that is equal to . Find  +  + 
0R  m

4. Four equal charges +q are placed at four corners of a square with its centre at origin and lying in
yz plane. The electrostatic potential energy of a fifth charge +q varies on x – axis as

APNI KAKSHA 36
(Physics)

(A) (B)

(C) (D)
5. Two identical thin rings, each of radius R meter are coaxially placed at distance R meter apart. If
Q1 and Q 2 coulomb are respectively the charges uniformly spread on the two rings, the minimum
work done in moving a charge q from the centre of one ring to that of the other is

(A) zero (B)


q ( Q1 − Q2 ) ( 2 −1 )
( 2.40R )
(C)
q 2 ( Q1 + Q2 )
(D)
q ( Q1 − Q2 ) ( 2 +1 )
40R ( 2.40R )
6. In a regular polygon of n sides, each corner is at a distance r from the centre. Identical charges are
placed at (n – 1) corners. At the centre, the intensity is E and the potential is V. the ratio V/E has
magnitude

(A) r n (B) r (n – 1) (C)


( n − 1) (D)
r ( n − 1)
r n
7. A uniform electric field having strength E is existing in x – y plane as shown in figure. Find the p.d.
between origin O and A (d, d, 0)

(A) Ed ( cos  + sin ) (B) −Ed ( sin  − cos  )

(C) 2Ed (D) None of these

APNI KAKSHA 37
(Physics)

8. Uniform electric field of magnitude 100V/m in space is directed along the line y = 3 + x . Find the
potential difference between point A(3, 1) and B(1, 3)

(A) 100V (B) 200 2V (C) 200V (D) 0


9. The dipole moment of a system of charge +q distributed uniformly on an arc of radius R subtending
an angle  / 2 at its centre where another charge –q is placed at centre
2 2qR 2qR qR 2qR
(A) (B) (C) (D)
   
10. Two sphere of radii R 1 and R 2 , joined by a fine long wire, are raised to a potential V. if 1 and  2

R1 3
represent the respective surface charge densities of the spheres, then for = we have
R2 4

(A) (B)

(C) (D) None of these


11. Two point charges (Q each) are placed at (0, y) and (0, -y). A point charge q of the same polarity
can move along X – axis. Then
y
(A) the force on q is maximum at x = 
2
(B) the charge q is in equilibrium at the origin
(C) the charge q performs an oscillatory motion about the origin
(D) for any position of q other then origin the force is directed away from origin
12. Three equal point changes (Q) are kept at the three corners of an equilateral triangle ABC of side
a. P is a point having equal distance a from A, B and C. if E is the magnitude of electric field and V
is the potential at point P, then
3Q 6Q 3Q 3 6Q
(A) E= (B) E = (C) E= (D) E =
40a 2 40a 2 40a 40a 2

APNI KAKSHA 38
(Physics)

13. Two concentric rings of radii R1 = 6m and R 2 = 4m are placed in y – z plane with their centres
at origin. They have uniform charge –q and +Q = 2 2q on the inner and outer rings respectively.

Consider the electrostatic potential to be zero at infinity. Then

(A) The electic potential is zero at origin


(B) The electric field intensity is zero at r = 2m
(C) A positive charged particle disturbed from origin along the x – axis will restore back to origin
(D) Where potential is maximum on the x – axis, field intensity is zero
14. Two particles A and B, each carrying a charge Q, are held fixed with a separation d between them.
A particle C having a mass m and charge –q, in kept at the middle point of line AB.
It displaced through a distance x perpendicular to AB. Assume x << d. Then :
(A) Force experienced by C is proportional to x
(B) Force experienced by C is proportional to d
1/2
 m30d3 
(C) Particle C may executes SHM with time period  
 qQ 
1/2
 m20d2 
(D) Particle C may executes SHM with time period  
 qQ 
15. A point charge +q is placed on the axis of a closed cylinder of radius R and height 25R/12 as
xq
shown. If electric flux coming out from the curved surface of cylinder is , then calculate x.
100

16. Two point electric dipoles with dipole moments ' P1 ' and 'P2 ' are separated by a distance ‘r’ with

their dipole axes mutually perpendicular as shown. The force of interaction between the dipoles
 1 
 where, k = 
 40 

APNI KAKSHA 39
(Physics)

2kPP
1 2 3kP1P2 4kPP
1 2 6kPP
1 2
(A) (B) (C) (D)
r4 r4 r4 r4
17. A dipole is placed at origin of co-ordinate system as shown in figure. Electric field at point P (0,
y) is given as

(A)
3y3
(
K ˆ ˆ
−i − j ) (B)
y3
(
K ˆ
−i + 2 ˆj) (C)
y3
(
K ˆ ˆ
−i − 2 j ) (D)
2y3
(
K ˆ ˆ
)
−i − 2 j

18. Two fixed charges −2Q and Q are located at the point of co-ordinates ( −3a, 0 ) and ( 3a, 0 )

respectively in (x – y) plane. Then all the points in x – y plane where potential is zero lies on a
(A) straight line parallel to x-axis (B) straight line parallel to y-axis
(C) a circle of radius 4a (D) circle of radius 2a
19. Two free charges +Q and +4Q are placed at a separation L. The system to stay in equilibrium.
L 2L
(A) The location of 3rd charge is at from Q are for 4Q
3 3
2L L
(B) The location of 3rd charge is at from Q are for 4Q
3 3
4Q
(C) q = −
q
4Q
(D) q = +
q
20. A small charged particle of mass m and charge q is suspended by an
insulated thread in front of a very large sheet of charge density  . The angle
made by the thread with the vertical, in equilibrium is

 q   q 
(A) tan −1   (B) tan −1  
 20mg   0mg 
 2q 
(C) tan −1   (D) zero
 0mg 
APNI KAKSHA 40
(Physics)

21. A particle of charge + q and mass ‘m’ moving under the influence of a uniform electric field E iˆ

and uniform magnetic field B kˆ follows a trajectory from P to Q as shown in figure. The velocities

at P and Q are v iˆ and −2 ˆj . Which of the following statement(s) is/are correct?

3  mv2 
(A) E= 
4  qa 

3  mv3 
(B) Rate of work done by the electric field at P is
4  a 
(C) Rate of work done by the electric field at P is zero
(D) Rate of work done by both the fields at Q is zero
22. In the given diagram three concentric conducting charged spherical shells are indicated. Initially
both the switches are open. Select the correct alternatives (s)

Q
(A) If only switch S 2 is closed then the charge transferred through this switch will be
2
Q
(B) If only switch S 2 is closed then the charge transferred through this switch will be
3
(C) If only switch S1 is closed then the charge transferred through this switch will be 2Q

Q
(D)If only switch S1 is closed then the charge transferred through this switch will be
2

APNI KAKSHA 41
(Physics)

EXERCISE-4
1. In space of horizontal EF (E = (mg)/q) exist as shown in figure and a mass m attached at the end
of a light rod. If mass m is released from the position shown in figure, the angular velocity of the

g
rod when it passes through the bottom most position is . Find  + 

2. Two identical particles of mass m carry a charge Q each. Initially one is at rest on a smooth
horizontal plane and the other is projected along the plane directly towards first particle from a
large distance with speed  . The closest distance of approach is

1 Q2 1 4Q2 1 2Q2 1 3Q2


(A) (B) (C) (D)
40 m 40 m2 40 m2 40 m2
3. A charged particle of charge Q is held fixed and another charged particle of mass m and charge q
(of the same sign) is released from a distance r. The impulse of the force exerted by the external

mQq
agent on the fixed charge by the time distance between Q and q becomes 2r is . Find
0r
+
4. The diagram shows three infinitely long uniform line charges placed on the X, Y and Z axis. The
work done in moving a unit positive charge from (1, 1, 1) to (0, 1, 1) is equal to

(A)
(  ln 2) (B)
(  ln 2) (C)
( 3 ln 2) (D) None
20 0 20

APNI KAKSHA 42
(Physics)

5. A wheel having mass m has charges +q and –q on diametrically opposite points. It remains in
mg
equilibrium on a rough inclined plane in the presence of uniform vertical electric field E = .
nq
Find n?

6. The figure shows charged spherical shells A, B and C having charge densities  , − ,  and radii
a, b , c respectively. If VA = VC , then b equal to

(A) a + c (B) a2 + c2 (C) ac (D) c – a


7. A charged particle moves with a speed  in a circular path of radius r around a long uniformly
charged conductor. Then
1 1
(A)   r (B)  (C)  (D)  is independent of r
r r
8. A fixed point charge Q is at origin. At t = 0, a charge q with mass m is at x = a with leftward velocity
kQq
V0 which satisfies = 3mV02 . The particle turns around and starts to move rightward at the
a
position ba.
b 6 b 3
(A) The ratio is (B) The ratio is
a 7 a 7
(C) The velocity of the particle at a large distance from the origin is 7V0
(D) The velocity of the particle at a large distance from the origin is zero

APNI KAKSHA 43
(Physics)

9. You are moving a negative charge q  0 at a small constant speed away from a uniformly charged
non – conducting spherical shell on which resides a negative charge Q. the electrostatic field of Q

is E. let U be the total energy of the system, Wa the work done by the force Fa you exert on q and
WE the work done by the electrostatic force FE on q. then, as q is being moved
(A) Wa = − WE , therefore U remains constant
(B) Fa = −FE
(C) U increases
(D) U decreases

10. S is a solid neutral conducting sphere. A point charge q of 1 10−6 C is placed at point A. C is the
centre of sphere and AB is a tangent. BC = 3m and AB = 4m.

(A) the electric potential of the conductor is 1.8 kV


(B) the electric potential of the conductor is 2.25 kV
(C) the electric potential at B due to induced charges on the spheres is -0.45 kV
(D) the electric potential at B due to induced charges on the spheres is 0.45 kV
11. Two infinite, parallel, non – conducting sheets carry equal positive charge denstiy  . One is placed
in the yz plane and the other at distance x = a. take potential V = 0 at x = 0

(A) For 0  x  a , potential Vx = 0



(B) For x  a , potential Vx = − (x − a)
0

(C) For x  a , potential Vx = (x − a)
0

(D) For x  0 , potential Vx = x
0
12. Small identical balls with equal charges are fixed at the vertices of a regular polygon of N sides,
each of length d. at a certain instant, one of the ball is released. After a long time interval, the
adjacent ball to the previous one is released. The difference in kinetic energies of the two released
balls is K at a sufficiently long distance from the polygon
APNI KAKSHA 44
(Physics)

(A) Final kinetic energy of the first ball is greater than of the second ball
(B) Final kinetic energy of the second ball is greater than that of the first ball
(C) Charge on each ball is 20dK
(D) Charge on each ball is 40dK
13. Two positive point charges each of magnitude 10C are fixed at positions A and B at a separation

2d = 6m . A negatively charged particle of mass m = 90 gm and charge of magnitude 10  10−6 C


is revolving in a circular path of radius 4m in the plane perpendicular to the line AB and bisecting
the line AB. Neglect the effect of gravity. Find the angular velocity of the particle. If gravity is also
considered will it still move in the circular path assuming AB to be horizontal.

(A) 100 rad/s (B) 200 rad/s (C) 300 rad/s (D) 400 rad/s
14. A square loop of side ‘ ’ having uniform linear charge density ‘  ’ is placed in ‘xy’ plane as shown
a
in figure. There is a non – uniform electric field E= ( x + ) ˆi where a is a constant. Find the
resultant electric force on the loop if = 10cm , a = 2N / C and charge density  = 2C / m

(A) 2  10−6 N (B) 4  10−6 N (C) 6  10−6 N (D) 8  10−6 N


15. Find the magnitude of uniform electric field E in N/C (direction shown in
figure) if an electron entering with velocity 100m/s making 30 comes
m
out making 60 (see figure), after a time numerically equal to of
e
electron.
(A) 25 (B) 50 (C) 75 (D) 100

APNI KAKSHA 45
(Physics)

2
16. A small ball of mass 1kg and charge C is placed at the centre of a uniformly charged sphere of
3
1
radius 1m and charge mC . A narrow smooth groove is made in the sphere from centre to surface
3
as shown in figure. The sphere is made to rotate about its vertical diameter at a constant rate of
1
revolutions per second. Find the speed w.r.t. ground with which the ball slides out from the
2
groove. Neglect any magnetic force acting on ball.

(A) 1 m/sec (B) 2 m/sec (C) 3 m/sec (D) 4 m/sec


17. In given diagram, there is a conducting sphere of radius r1 which is surrounded by dielectric (r )

. If conducting sphere is given charge ‘q’ then surface density of polarization charges on outer
surface of dielectric layer is

(A)
r q
(B)
q
(C)
(r −1) q (D)
(r −1) q
4r22 4r r22 4r2
2 4r r22
18. Three particles, each of mass 1g and carrying a charge q are suspended from a common point by
three insulated massless strings, each 100 cm long. If the particles are in equilibrium and are
located at the corners of an equilateral triangle of side length 3 cm, calculate the charge q on each

(
particle. Take g = 10 ms−2 . )
−9 −9 −9 −9
(A) 1.17 10 C (B) 2.17 10 C (C) 3.17 10 C (D) 4.17 10 C

APNI KAKSHA 46
(Physics)

19. Two identical positive point charges, each having a charge Q are fixed at a separation 2a. A point
charge q lies midway between the fixed charges. For small
(1) displacement (compared to a) along the line joining the fixed charges, the charge q executes
SHM, if it is positive in nature.
(2) lateral displacement, the charge q executes SHM, if it is negative in nature. The ratio of periods
of oscillations in the above two cases.
1 1 1 1
(A) (B) (C) (D)
2 3 2 3
20. A charge +q is fixed at each of the points x = x 0 , x = 3x 0 , x = 5x 0 ,…….,  on the x – axis and a

charge –q is fixed at each of the points x = 2x 0 , x = 4x 0 , x = 6x 0 ,…….,  . Here, x 0 is a positive

Q
quantity. Take the electric potential at a point due to charge Q at a distance r from it to be .
40r
Then, the potential at origin due to the above system of charges is
q qln 2
(A) 0 (B)  (C) (D)
80x0 ln2 40 x 0
21. Consider an imaginary hemispherical surface. A semi–infinite wire of charge density  is kept
with one of its end coinciding with center of hemisphere and wire is kept along the symmetric
axis of the hemisphere as shown in the figure. The electric flux through the spherical surface
(Curved surface) of the hemisphere is

λR λR 3λR λR
(A) (B) (C) (D)
30 40 40 20

APNI KAKSHA 47
(Physics)

EXERCISE-5
1. A small electric dipole is placed at origin with its axis being directed along the positive x – axis.

The direction of electric field due to the dipole at a point (1m, )


2m,0 is along the
(A) z – axis (B) y – axis (C) x – axis (D) line y = x
2. Two spherical, non – conducting, and very thin shells of uniformly distributed positive charge Q
and radius d are located a distance 10d from each other. A positive point charge q is placed inside
one of the shells at a distance d/2 from the centre, on the line connecting the centres of the two
shells, as shown in the figure. What is the net force on the charge q?

qQ qQ
(A) to the left (B) to the right
3610d2 3610d2
362qQ 360qQ
(C) to the left (D) to the right
3610d2 3610d2
3. A bullet of mass m and charge q is fired towards a solid uniformly charged sphere of radius R and
total charge +q. If it strikes the surface of sphere with speed u, the minimum speed u so that it can
q
penetrate through the sphere is . Find  +  ?
0mR

(Neglect all resistance forces or friction acting on bullet except electrostatic forces)
4. There are four concentric shells A, B, C and D of radii a, 2a, 3a and 4a respectively. Shells B and D
are given charges +q and –q respectively. Shell C is now earthed. The potential difference VA − VC

Kq
is . Find n?
na
5. The potential energy of a system of two identically charged hollow spheres (as shown in the figure)

q  1 1 
is equal to − . find  +  ?(Assume the charge distribution to be uniform)
0  R r 
APNI KAKSHA 48
(Physics)

6. If charges q/2 and 2q are placed at the centre of face and at the corner of a cube, then the total flux
q
through the cube is . Find 
0
7. A large insulating thick sheet of thickness 2d carries a uniform charge per unit volume . A
particle of mass m, carrying a charge q having a sign opposite to that of the sheet, is released from
the surface of the sheet. The sheet does not offer any mechanical resistance to the motion of the
particle. Find the oscillation frequency  of the particle inside the sheet.

1 q 1 2q 1 q 1 2q
(A)  = (B)  = (C)  = (D)  =
2 m0 2 m0 4 m0 4 m0
2
8. Figure shows a metal ball of mass 50kg and radius m is placed on an insulating uncharged

stand. In space an upward electric field 5 105 N / C is switched on. A stream of light ions is
incident on the ball from left side at a speed 2 106 m / s as shown in figure. If charge on ball at t
= 0 was zero, find the time at which ball will be lifted from the stand. The charge density of ion

beam is 5  10 coulomb m . Assume that all charge incident on the ball is absorbed.
−12 3

(A) 5sec (B) 15 sec (C) 25 sec (D) 35 sec


9. N identical charges each of charge ‘q’ are symmetrically placed on the circumference of circle of
radius ‘r’. Four adjacent charges are slowly taken to infinity in the sequence a shown in figure. If
W1 , W2 , W3 and W4 is the respective work done in slowly displacing q1 , q 2 , q3 and q 4 . If

 
 −q 2 
( W3 − W4 ) is x 
3 
then ___
 4t 0 r sin 
 n 
APNI KAKSHA 49
(Physics)

(A) 0.20 (B) 0.50 (C) 0.71 (D) 0.82


10. Two equal charges Q are at opposite corners of a square of side ‘a’ and an electric dipole of
moment p is at a third corner, pointing towards one of the charges. If p = 2 2a Q; The field
strength at the fourth corner of the square is :-

17 / 2 17 / 4 Q 3
(A) Q (B) Q (C) (D) Q
40 a 2 40 a 2 4 20 a 2 40 a 2
11. Consider a thin conducting shell of radius R carrying total charge Q. Two point charges Q and 2Q
R
are placed on points A and B, which are at a distances of and 2R from the center of the cell
2
respectively as shown in the figure. If the cell is earthed how much charge will flow to the earth?

Q
(A) 3Q (B) Q (C) 2Q (D)
2
12. Two hemispherical thin shells made of insulating materials are concentrically
arranged in a free space as shown. The radii of the smaller and larger
hemispheres are ‘R’ and ‘2R’ and they carry positive charges ‘Q’ and ‘2Q’
respectively. The charges are uniformly distributed over the surfaces of the
shells. Then choose the correct option(s).
Q2
(A) The electrostatic force between the hemispherical shells is
80 R 2

Q2
(B) The electrostatic force between the hemispherical shells is
160R 2
Q
(C) The net electric potential at the centre ‘O’ of the shells is
20R
3Q
(D) The net electric potential at the centre ‘O’ of the shells is
20R
APNI KAKSHA 50
(Physics)

13. A charged fixed annular disc of uniform positive charge density  has inner and outer radii R
and 2R respectively. A negatively charged particle of mass m and charge –q is released on the axis
of the disc from a distance x from the centre. Then

(A) The speed of the particle when reaches the centre of ring,

v=
q
m0 (
R + R 2 + x 2 − 4R 2 + x 2 )
(B) The speed of the particle when it reaches the centre of ring,

v=
q
m0 (
R + 4R 2 + x 2 − R 2 + x 2 )
(C) Work done by electrostatic force till the particle reaches the centre of ring

=
q
20
(
R + R 2 + x2 − 4R 2 + x2 )
1 q
(D) If x << R, the frequency of oscillation of the particle is f =
2 40mR
14. A semi-circular disc of mass M and radius R with linear charge density  on its curved
circumference is hinged at its centre and placed in a
uniform electric field as shown in the figure. Match the
statements from List I with those in List II and select the
correct answer using the code given below the lists.

List – I List – II
(A) The net force acting on the ring is (P) M

E
(B) If the ring is slightly rotated about O and released, (Q) RE
find its time period of oscillation
(C) The work done by an external agency to rotate it (R) 2K
E−
through an angle  is R
(D) Magnitude of electric field at ‘O’ will be (S) 
4ER 2 sin2  
2

APNI KAKSHA 51
(Physics)

A B C D
(A) P Q R S
(B) Q S P R
(C) Q P S R
(D) R S P Q
15. An infinite non-conducting plate with uniform charge density  is kept parallel to yz plane and

at a distance ‘d’ from a dipole p which itself is located at the origin. An equipotential surface for
1/3
 p 
this system is spherical, centred at origin, having radius R(<d). Given that R =   ; find the
 n 
integer n.

16. A small ball of mass 1 kg and charge 8 C is placed at the centre of a uniformly charged sphere
1
of radius 2m and charge mC . A narrow smooth groove is made in the sphere from centre to the
6
surface of sphere and the sphere is made to rotate about its vertical diameter at a constant rate
1
of revolution per second. Find the speed (in m/s) with respect to ground with which the ball
4
slides out from the groove. Ignore any magnetic force acting on the ball. (Take 2 = 1.41 )
(A) 2.82 (B) 1.82 (C) 0.82 (D) 3.82
17. An infinite non-conducting uniformly charged sheet has a hole of radius ‘R’ in it (change density
=  ). An electron is placed on an axis passing through the centre of hole and perpendicular to
the plane of sheet at a distance ‘R’ from the centre. The speed with which the electron reaches
the centre is ….
(   0) Given : electric field on the axis of a uniformly charged disc of radius ‘r’

  x 
E(x) = 1 − 2 2 
2 x +r 

(A)
eR
(B)
2eR
(C)
( )
2 −1 eR
(D)
2eR
m0 m0 m0 m0

APNI KAKSHA 52
(Physics)

18. If electric force ( F) on a point charge ‘q’ due to another charge ‘Q’ obeys following law
F=
(
Qq 1 − r r )
40 r 3

Where  ; positive constant, r is position vector of charge ‘a’ relative to ‘Q’.

(A) Electric field to point charge Q is E =


(
Q 1 − r )r
40 r 3

(B)  Ed over a closed path will be equal to zero

qenclosed
(C) Gauss’s law  E  ds = 0
holds true

(D) All statements are correct


19. A conducting sphere of radius R and charge Q is placed near a uniformly charged non-conducting
infinitely large thin plate having surface charge density  . Then find the potential at point A (on
 1 
the surface of sphere) due to charge on sphere  here K = , 0 = 
 40 3

Q  Q  Q
(A) K − R (B) K − (C) K (D) None of these
R 4 0 R 2 0 R
20. A segment of a charged wire of length , charge density 1 , and an infinitely long charged wire,

charge density 1 , lie in a place at right angles to each other. The separation between the wires

is r0 . Determine the force of interaction between the wires.

12   12  
(A) F = loge 1 +  (B) F = loge 1 + 
20  r0  40  r0 
212   412  
(C) F = loge 1 +  (D) F = loge 1 + 
0  r0  0  r0 

APNI KAKSHA 53
(Physics)

21. A charge particle ‘q’ lies at the center of two concentric hollow spheres of inner radii R and 3R
and outer radii 2R and 4R respectively. What amount of work has to be performed to slowly
transfer the charge ‘q’ from center through the orifice to infinity.

5 q2 q2 7 q2 q2
(A) (B) (C) (D)
96 πε0R 16πε0 R 96 πε0R 96πε0R
22. A dumbbell like structure is made by affixing two particles A and B at the ends of a light non
conducting spring. Both the particles have equal mass m and the particle B carries a positive
charge q. A uniform electric field of intensity E pointing in the negative x – direction is
established in the region x > 0 and gravity is absent everywhere. The dumbbell is initially at rest
on the x – axis in the region x < 0. It is projected with a velocity u in the positive x – direction as
shown in the figure. After a while, the dumbbell is observed moving in the negative x – direction
with the same speed u . During this interval, particle A never enters the region of electric field
and the spring length becomes minimum only once. How much time the particle B spends in the
electric field? (spring is in its natural length before entering and after coming of the electric field)

mu 2mu mu 4mu
(A) (B) (C) (D)
2qE qE 4qE qE
23. There is an infinite line of uniform linear density of charge +λ . A particle of charge ‘-q’ and mass
‘m’ is projected with initial velocity v 0 at an angle θ with the

line of charge from a distance ‘a’ from it. The speed of the
particle is found to be minimum when its distance from the line
( Kmε v sin θ/qλ )
2 2

of charge is ae .Then the Value of K is


0 0

(A)  (B) 2 (C) 3 (D) 4

APNI KAKSHA 54
(Physics)

24. Two long wires have uniform charge density  per unit length each. The wires are non-coplanar
and mutually perpendicular. Shortest distance between them is d. The interaction force between
them is:
2 2 22 2
(A) (B) (C) (D)
20 20 0 4 0

25. A frustum is cut from a right circular cone. The two circular faces have radii R and 2R and
their centers are at O1 and O2 respectively. Height of the frustum is h = 3R. When a point charge

Q is placed at O1 , the flux of electric field through the circular face of radius 2R is 1 and when

the charge Q is placed at O2 , the flux through the other circular face is  2 . Then

1  13 − 3  10 1  13 − 3  13
(A) =  (B) = 
2  10 − 3  13 2  10 − 3  10

1  10 − 3  10 1  13 − 6  10
(C) =  (D) = 
2  13 − 3  13 2  10 − 6  13
26. A non-conducting sphere of radius ‘a’ and unit permittivity has a surface charge density ‘  ’
varying with polar angle ‘ θ ’ as  =  0 cosθ [  0 is a positive constant]. Intensity of electric field

at ( r , ) where r  a is given by

 0 cosθ 0 r 0  0 r cosθ
(A) (B) (C) (D)
30 3 0 a 3 0 3 0 a
27. The figure shows the variation of electrostatic potential V in volt with the distance of position of
point along x – axis from origin due to continuous volume charge distribution. In the region x = -

( )
1 m to X = + 1m, the graph is parabolic V = 15 − 5x2 and rest portion of graph is straight line.
−12 −1 −2 2
Choose the correct option(s) (  0  permittivity of free space = 8.85 10 N m C ). The

direction of E along Positive x – axis is considered as positive.

APNI KAKSHA 55
(Physics)

(A) This graph of potential may be due to a thick sheet of infinite dimension
( −1m  x  1m, −  y  and −  z  ) with constant volume charge density
1.77 10−10 C / m3
(B) This graph of potential may be due to a thick sheet of infinite dimension
( −1m  x  1m, −  y  and −  z  ) with constant volume charge density
0.885 10−10 C / m3
(C) The variation of electric field with the distance due to a thick sheet of infinite dimension
( −1m  x  1m, −  y  and −  z  ) may be

(D) The variation of electric field with the distance due to a thick sheet of infinite dimension
( −1m  x  1m, −  y  and −  z  ) may be

28. ( )
The electric field in the space is given by E = E0 xi + y j + zk . Consider a right circular cylindrical

surface whose radius is ‘a’ and height ‘ h ’. Now choose the correct option(s).

APNI KAKSHA 56
(Physics)

(A) The electric flux through lower circular base is zero


(B) The electric flux through upper circular top is  a 2 hE0

(C) The electric flux through lateral surface is 2 a 2 hE0

(D) The total electric flux through cylindrical surface is 4 a 2 hE0

29. A charged hemispherical shell has a uniform charge density  . Calculate the ratio of electric field
due to shell at A to that at B

APNI KAKSHA 57
(Physics)

Proficiency Test-1
1. A, B, C, D, P and Q are points in a uniform electric field. The potentials at these points are
V(A) = 2 volt
V(P) = V(B) = V(D) = 5 volt
V(C) = 8 volt
The electric field at P is
−1
(A) 10Vm along PQ (B) 15 2Vm−1 along PA
(C) 5Vm−1 along PC (D) 5Vm−1 along PA
2. 4 charges are placed each at a distance a from origin. The dipole moment of configuration is

(A) 2qajˆ (B) 3qajˆ (C) 2aq ˆi + ˆj (D) none of these

3. Three concentric metallic spherical shell A, B and C or radii a, b and c (a < b < c) have surface
charge densities − , + , and − respectively. The potential of shell A is
 
(A)    a + b − c (B)    a − b + c
 0   0 

(C)    b − a − c (D) None of these
 0 
4. Two identical small conducting spheres, having charges of opposite sign, attract each other with a
force of 0.108N when separated by 0.5m. The spheres are connected by a conducting wire, which
is then removed, and thereafter, they repel each other with a force of 0.036N. the initial charges on
the spheres are

(A) 5  10−6 C and 15 10−6 C (B) 1.0 10−6 C and 3.0 10−6 C

(C)  2.0 10−6 C and 6.0 10−6 C (D) 0.5 10−6 C and 1.5 10−6 C
5. A solid sphere of radius R is charged uniformly. At what distance from its surface is the
electrostatic potential half of the potential at the centre?
R R
(A) R (B) (C) (D) 2R
2 3
APNI KAKSHA 58
(Physics)

6. n small conducting drops of same size are charged to V volts each. If they coalesce to form a single

large drop, then its potential is Vn / . Find  +  ?


7. The system of charges as shown in the figure. A thick spherical shell with an inner radius a and an
outer radius b is made of conducting material. A point charge +Q is placed at the centre of the
spherical shell and a total charge –q is placed on the shell. Charge –q is distributed on the surfaces
as

(A) –Q on the inner surface, -q on outer surface


(B) –Q on the inner surface, -q + Q on outer surface
(C) +Q on the inner surface, -q - Q on outer surface
(D) The charge –q is spread uniformly between the inner and outer surface
8. A charge –Q is uniformly distributed over a non – conducting semicircular rod of radius R. the
potential at the centre is

1 ( −Q) 1 Q 1 2Q
(A) 0 (B) . (C) . (D) .
40 R 40 2R 40 R
9. Two point charges +q and –q are held fixed at points (-d, 0) and (d, 0), respectively, of a (x, y)
coordinate system. Then

(A) the electric field E at all points on the x – axis has same direction
(B) E at all points on the Y – axis is along ˆj
(C) No work is done in bringing a test charge from infinity to the origin

(D) The dipole moment is 2qd directed along î


10. A positive point charge is placed inside a spherical metallic shell. The electric field lines are
represented by

(A) (B) (C) (D)

APNI KAKSHA 59
(Physics)

11. In the electric field due to a point charge q, a test charge is carried from A to the points B, C, D and
E lying on the same circle around q. The work done is
(A) the least along AB
(B) the least along AD
(C) zero along any one of the paths AB, AD, AC and AE
(D) the least along AE
12. Consider the situation depicted in the adjacent figure. The work done in taking a point charge from
P to A is WA , from P to B is WB and from P to C is WC . Therefore

(A) WA  WB  WC (B) WA  WB  WC

(C) WA = WB = WC (D) WA  WB  WC

13. In the figure, the ratio of electic field at point P to that at point Q will be (r = 2 ) .

5
(A) 2:1 (B) 5: (C) 20 5 : 9 (D) 5 5 : 3
3
14. The smooth circular pipe is kept fixed in a horizontal plane. If mass m with charge +q is slightly
displaced from the position shown, then distance travelled by the ball when the magnitude of
normal becomes minimum would be

APNI KAKSHA 60
(Physics)

 2 −1 5
(A) R cos−1   (B) R cos 1/ 3 (C) R tan −1 (D) R
 3 2
15. A uniformly charged square plate having side L carries a uniform surface charge density  . The
plate lies in the y-z plane with its center at the origin. A point charge q lies on the x-axis. The flux
of the electric field of q through the plate is  0 ; while the force on the point charge q due to the

plate is F0 , along the x-axis. Then,

F0 F0 F0L 0
(A)  = (B)  = (C)  = (D)  =
0L 0 0 F0
16. A point charge ‘q’ is placed at distance ‘a’ from the centre of an unchanged thin spherical
conducting shell of radius R = 2a. A point ‘P’ is located at a distance ‘4a’ from the centre of the
conducting shell as shown. The electric potential due to induced charge on the inner surface of
the conducting shell at point ‘P’ is

kq 5kq kq kq
(A) − (B) − (C) + (D) −
a a 5a 5a
17. An electric dipole is constructed by fixing two circular charged rings, each of radius a, with an
insulating contact, one of these rings has a total charge +Q and the other has total charge –Q. If
the charge is distributed uniformly along each ring, the dipole moment about the point of contact
ˆ / 4 , n = _________.
will be n QaZ

APNI KAKSHA 61
(Physics)

Proficiency Test-2
1. A charge Q has to be divided between two solid spheres of radii R which are at distance d from

each other ( d  R ) . What should be the value of charge, which we should be placed on the
spheres, so that the force of attraction between them is maximum?

Q 3Q Q 2Q Q Q Q 4Q
(A) , (B) , (C) , (D) ,
4 4 3 3 2 2 5 5
2. An insulating rod of uniform linear charge density  and uniform linear mass density  lies on a
smooth table whose surface is xy – plane. A uniform electric field E is switched on

(A) If electric field is along x – axis, the speed of the rod when it has travelled a distance d is

2Ed

(B) If electric field E is at an angle  (  90 ) with x – axis, the speed of the rod when it has

2Edcos 
travelled a distance d is

(C) Torque on the rod due to the field about centre of mass in case B is into the plane of paper
(D) Torque on the rod due to the field about centre of mass in case B is zero
3. The arrangement shown consists of three elements
(a) a thin rod of charge −3.0 C that forms a full circle of radius 6.0 cm
(b) a second thin rod of charge 2.0 C that forms a circular arc of radius
4.0 cm and concentric with the full circle, subtending an angle of 90 at
the centre of the full circle
(c) an electric dipole with a dipole moment that is perpendicular to a radial line and has

magnitude 1.28  10 C − m.
−21

Find the net electric potential in volts at the centre.


(A) 0V (B) 1V (C) 2V (D) 3V

APNI KAKSHA 62
(Physics)

4. Figure shows a irregular space conductor with irregular cavity inside it. A charge Q is placed
inside cavity and a charge Q’ is placed outside conductor. Let Q'ind be charge induced at outside

surface of conductor and Qind be the charge induced at inside surface of cavity. A ‘B’ is an

arbitrary line passing through charge Q and a, b, c be the points on line as shown in figure. Let
E, E ', Eind and E 'ind represent electric field at different points due to charge Q, Q' Qind , & Q'ind

respectively. If Va , Vb and Vc represent potential at point a, b, and c. Choose the incorrect

statement

(A) Va = Vb = Vc
r0

(B) Vb =  ( E + E'+ E
−
ind )
+ E'ind  dr , where rb = position vector of point b

rb

(C) Vb =  ( E'+ E' )  dr


−
ind

(D) At point ‘b’, E ' = 0

5. A cavity is made inside a solid conducting sphere and a charge q is placed inside the cavity at the
centre. A charge q1 is placed outside the sphere as shown in the figure. Point A is inside the

sphere and point B is inside the cavity. Then

(A) Electric field at point A is zero


1 q
(B) Electric field at point B is
40 y2
1  q1 q 
(C) Potential at point A is +
40  a R 

1  q1 q q q 
(D) Potential at point B is + + −
40  a R y r 

APNI KAKSHA 63
(Physics)

6. A disc having uniform surface charge density in upper part + and − in lower half placed on
rough horizontal surface as shown in figure. A uniform electric field is set up as shown if mass of
disc is M and its radius is R as well sufficient friction is present to prevent slipping. The
acceleration of disc is

8 R 2E0 9 R 2E0 4 R 2E0 3 R 2E0


(A) (B) (C) (D)
9 M 8 M 3 M 4 M
7. Two identical thin rods of length 2a carry equal charges +Q that is distributed uniformly along
their lengths. The rods lie along the x-axis with their centers separated by a distance b > 2a. The
Q2  b2 
magnitude of the force exerted by the left rod on the right one is F = log e  2 2
.
0a 2  b −a 
Find  +  ?

8. Two spherical shells of inner radius R and 3R are placed far from each other. They are made of
some thin conducting material, the width of their wall d is thin: d≪R. At the centres of the
spheres there are charges of 2Q and Q respectively. What is the minimum work which should be
done in order to interchange the charges? (There are small holes on the walls.)
5d Q2 d Q2 d Q2 d Q2
(A) (B) (C) (D)
6πε0 R 2 4πε0 R 2 8πε0 R 2 3πε0 R 2
9. Consider a cube as shown in the figure-I: with uniformly distributed charge in its entire volume.
Intensity of electrical field and potential at one of its vertex P are E0 and V0 respectively. A

portion of half the size (half edge length) of the original cube is cut and removed as shown in the
figure –II. Find modulus of electric field and potential at the point P in the new structure

APNI KAKSHA 64
(Physics)

E0 3V 3E0 V 3E0 7V 7E0 7V


(A) and 0 (B) and 0 (C) and 0 (D) and 0
2 4 4 2 4 8 8 8
10. Two-point charges are kept as shown in figure from a neutral conducting shell and P is a point
inside the shell. Find potential at P due to induced charges.

kq kq kq kq kq kq
(A) − (B) − (C) − (D) 0
d 2 2d d

APNI KAKSHA 65
(Physics)

Proficiency Test-3
1. A large sheet carries uniform surface charge density  . A rod of length 2 has a linear charge

density  on one half and − on the second half. The rod is hinged at mid point O and makes an
 
angle  with the normal to the sheet. The torque experienced by the rod is . Find  +  ?
0

2. Two short electric dipoles are placed as shown. The energy of electric interaction between these
dipoles will be

1 2 cos 
2kPP −2kPP
1 2 cos  −2kPP
1 2csin  −4kPP
1 2 cos 
(A) (B) (C) (D)
r3 r3 r3 r3
3. A small block of mass m is kept on a smooth inclined plane of angle 30 placed in an elevator going
upward with acceleration a. electric field E exists between the vertical sides of the wall of the
Elevator. The charge on the block is +q. the time taken by the block to come to the lowest point of

h
the inclined plane is 2 . Find  +  ?
gE
(g + a ) − 
m
(Take the surface to be smooth)

APNI KAKSHA 66
(Physics)

4. An ideal dipole of dipole moment P is placed in front of an uncharged conducting sphere of radius
R as shown.

KP
(A) the potential at point A is
( r − R )2
KP
(B) The potential at point A is
r2
KP
(C) the potential due to dipole at point B is
( r + R )2
KP
(D) The potential due to dipole at point B is
r2
5. A thin conducting spherical shell of radius ‘R’ is given charge ‘2q’. Two point charges ‘q’ and ‘3q’
are placed at distances 0.8R and 4R from the centre ‘C’ of the shell respectively as shown. If the
shell is earthed, find the charge (in C ) that will flow from conducting shell to the earth. (Given
q = 2C )

(A) 6.50 (B) 7.50 (C) 5.50 (D) 8.50


6. Two small balls A and B of positive charge Q each and masses m and 2m respectively are
connected by a non-conducting light rigid rod of length L. This system is released in a uniform
electric field of strength E as shown. Just after the release (assume no other force acts on the
system).

APNI KAKSHA 67
(Physics)

(A) rod has zero angular acceleration


QE
(B) rod has angular acceleration in anticlockwise direction
2mL
2QE
(C) acceleration of point A is towards right
m
QE
(D) acceleration of point A is towards right
m
6. On a semicircular ring of radius 4R, charge +3q is distributed in such a way that on one quarter

+ q is uniformly distributed and on another quarter +2q is uniformly distributed. Along its axis a
smooth non-conducting and uncharged pipe of length 6R is fixed axially as shown. A small ball of
mass m and charge + q is thrown from the other end of pipe. The ball can come out of the pipe if
(Neglect the effect of induction and also consider pipe to be friction less)

3q2 3q2 3q2 q2


(A) u < (B) u > (C) u  (D) u <
40πε0Rm 40πε0Rm 80πε0Rm 40πε0Rm
7. The figures below depict two situations in which two infinitely long static line charges of

constant positive line charge density  are kept parallel to each other. In their resulting electric
field, point, charges, q and –q are kept in equilibrium between them. The point charges are
confined to move in the x direction only. If they are given a small displacement about their
equilibrium positions, then the correct statement(s) is(are)

(A) Both charges execute simple harmonic motion.


(B) Both charges will continue moving in the direction of their displacement.
(C) Charge +q executes simple harmonic motion while charge –q continues moving in the
direction of its displacement.
(D) Charge –q executes simple harmonic moving in the direction of its displacement.

APNI KAKSHA 68
(Physics)

8. Two discs having uniform charge densities 1 and  2 respectively are placed parallel to each

other with their axis coinciding as shown. Flux of electric field linked with disc 1 is 1 and flux

of electric field linked with disc 2 is  2 . Then

(A) 1 = 2

1 2
(B) =
2 1
1 1
(C) =
2 2
(D) For any set of 1 and  2 , the relation 11 = 22 is always true

9. A small dipole having dipole moment p is placed in front of a fixed solid uncharged conducting
sphere as shown in the diagram.

kpcos 
(A) The net potential at point A lying on the surface of the sphere is
r2
kpcos2 
(B) The net potential at point A lying on the surface of the sphere is
r2
(C) The potential at point A lying on the surface due to induced charge will be
kpcos (cos  − 1)
r2
(D) The potential at point A lying on the surface due to induced charge will be
kpcos (1 − cos )
r2

APNI KAKSHA 69
(Physics)

10. A uniform dielectric hollow cylinder of mass M , radius R , length l carrying uniform charge of
surface charge density  can rotate without friction about a fixed horizontal axle that coincides
with the axis of the cylinder. Several turns of a light thin insulating cord are wrapped on the
cylinder and a block of mass m is suspended from the free end of the cord. Initially the block is
held at rest as shown in the figure. Find acceleration of the block after it is released. Neglect
charge transferred to the cord and fringing of magnetic field at the ends of the cylinder.
Acceleration due to gravity is g and permeability of the medium inside the cylinder is  0 choose

correct options.
mg
(A) Acceleration of the block just after release is .
m+μ0σ2 πR 2l
mg
(B) Acceleration of the block just after release is
m+M+μ0σ2 πR 2l
(C) Tension in the string just after release is zero
(D) Tension in the string just after release is non-zero
11. A small charged bead can slide on a circular frictionless, insulating wire frame. A point like dipole

is fixed at the center of circle; dipole moment is p . Initially the bead is on the plane of symmetry
of the dipole. Bead is released from rest. Ignore the effect of gravity. Select the correct options.

(A) Magnitude of velocity of bead as function of its angular position θ (wrt the axis of the dipole)

Qp(-cosθ)
is .
2πε0mr 2
(B) Normal force exerted by the wire frame on bead is zero.
(C) If the wire frame were not present bead executes circular motion and returns to initial point
after tracing a complete circle.
(D) Bead would move along a circle path until it reaches the opposite its starting position and
executes periodic motion.

APNI KAKSHA 70
(Physics)

EXERCISE-1
1 2 3 4 5 6 7 8 9 10
D B B A 4 A C 10 10 B
11 12 13 14 15 16 17 18 19 20
D B C B D 4 C A B D

EXERCISE-2
1 2 3 4 5 6 7 8 9 10
A D A D 100 B D B 6 D
11 12 13 14 15 16 17 18 19 20
A 2 C D 1 40 C 3 C 4

EXERCISE-3
1 2 3 4 5 6 7 8 9 10
B A 8 B B B A D A A
11 12 13 14 15 16 17 18 19 20
ABD BC BCD AC 7 B D C AC B
21 22
ABD AC

EXERCISE-4
1 2 3 4 5 6 7 8 9 10
3 B 5 B 2 D D AC BD AC
11 12 13 14 15 16 17 18 19 20
ABD AD D B D B D C A D
21
D

APNI KAKSHA 71
(Physics)

EXERCISE-5
1 2 3 4 5 6 7 8 9 10
B A 5 6 6 2 A C B A
11 12 13 14 15 16 17 18 19 20
A BC ACD C 2 A C AB A A
21 22 23 24 25 26 27 28 29 30
C D A A A C AC ABC 00.17

Proficiency Test-1
1 2 3 4 5 6 7 8 9 10
B A C B C 5 B B C C
11 12 13 14 15 16 17
C C C A B D 8

Proficiency Test-2
1 2 3 4 5 6 7
C AD A ABCD ABCD A 20
8 9 10
D A D

Proficiency Test-3
1 2 3 4 5 6
4 B 5 BC B D
7 8 9 10 11 12
C BD BC BD ABD

APNI KAKSHA 72

You might also like